You are on page 1of 88

Nervous system -DONE

MEDICINE
What is used to prevent delayed cerebral ischemia in pt with SAH ? /MOA
?
Nimodipine, a selective calcium channel blocker, improves outcomes in patients with
cerebral vasospasm by inducing cerebral vasodilation and decreasing calcium-
dependent excitotoxicity.

Cerebral vasospasm with subsequent ischemia is the most common complication of SAH


and typically manifests 3-12 days after the initial incident with a change in mental status
and/or new focal neurological deficits.

CT scan unchanged, unlike with rebleeding.

Cause of arterial vasospasm after SAH ?

due to the release of vasoconstrictive factors from damaged erythrocytes in the


subarachnoid space and the inability of damaged vascular endothelial cells to produce
vasodilators (eg, nitric oxide). > delayed cerebral ischemia

Rebleeding as a complication occurs in first 24 hours - use antihypertensives - eg ACE


inhib and labetalol

Medial medullary syndrome- ischemic stroke?


occlusion of the anterior spinal artery.

S/s : -

1. contralateral hemiparesis (due to damage of the lateral corticospinal tract),

2. contralateral hemisensory loss (dorsal column medial lemniscal pathway),

3.ipsilateral tongue paralysis (hypoglossal nucleus).

SAH complications ?
1.hydrocephalus

hours to days after SAH due to blood breakdown products obstructing the flow of


cerebrospinal fluid or clogging the arachnoid villi

neuroimaging > ventricular dilation.

2.Rebleeding

occurs early (<24 hours) after SAH - CT scan -newly extravasated blood.

Prevention - surgical repair of the aneurysm

3. Vasospasm

Nervous system -DONE 1


3-12 days after SAH > delayed cerebral ischemia (eg, sudden change in mental status,
new focal neurological deficits)- CT scan no new findings.

Prevention - initiation of nimodipine

Dx- CT angiography

tics are exacerbated by?


anxiety, fatigue, and psychosocial stress

Essential tremor - findings ?


tremor of the hands that is suppressed at rest - may not be complete

exacerbated by outstretched arms, and more pronounced at the end of goal-directed


movements

How does ET progress ? Is essential tremor inherited ?

slowly progressive and often hereditary; it can be passed in an autosomal dominant


fashion.

Dystonic tremor vs Essential tremor ?


Both have tremors that can worsen with the arms outstretched and vanish with rest

usually unilateral (often B/L - asymmetric)

task specific eg writing and typing ( irrespective of task - end of intended action)

Spinocerebellar ataxia vs Essential tremor ?


Both have intention tremor that worsens in amplitude at the end of the intended action

dysdiadochokinesis , dysmetria , scanning speech typically present ( ABSENT - NO


NEUROLOGICAL FINDING)

dysdiadochokinesis - inability to perform rapidly alternating movements , dysmetria -


overshooting on the finger-to-nose test , scanning speech - pauses between syllables of
words.

Define  mononeuritis multiplex ?
a neuropathy of ≥2 noncontiguous peripheral nerves.

Eg found in polyarteritis nodosa

Polyarteritis nodosa - Arterial damage tends to be most severe in the


following locations
1.Neurological system - Mononeuritis multiplex(70% pts) - damage to arteries supplying large
peripheral nerves - Most are asymmetric polyneuropathy

2.Skin: >livedo reticularis(reddish-blue, net-like pattern, as seen in this patient)


>erythematous nodules, and palpable purpura.

Nervous system -DONE 2


3.Renal system: Almost all patients with PAN have renal insufficiency due to renal artery
narrowing, thrombosis, and infarctions.

4.Gastrointestinal system: Damage to mesenteric arteries can cause abdominal pain,


mesenteric ischemia, and bowel perforation.

Case
53y man Rt leg and lt hand weakness + sensory loss , Cant flex wrist , Wt loss, intermittent
fever , ESR high , Hb 10g/dl , Cr - 2.2 mg/dl

Amyloidosis vs Polyarteritis nodosa ?

Skin - purpura particularly in a raccoon pattern around the eyes( Livedo reti , palpable
purpura , erythematous nodules)

Fever and ESR - non inflammatory so absent ( present )

what is spared in Polyarteritis nodosa? 


lungs are generally spared.

CSF analysis of Multiple sclerosis ?


1.Opening pressure - normal

2.appearance - normal

2.Total protein - normal + IgG relatively higher than other protein

3.cell count - normal with T lymphocyte predominance

Case
3 months ago trigeminal neuralgia , week ago viral infection , week ago b/l LE weakness ,
Decreased vibration and position in UE , DTR normal, dx ?

Multiple sclerosis neurologic deficits disseminated in space and time (eg, trigeminal


neuralgia, spastic lower limb paralysis, left upper limb sensory loss)

Can this be GBS?

Ascending paralysis unlike this pt where trigem - neuralgia was present months ago (
neuro deficits disseminated in time and space)

DTR reduced/absent ( DTR normal )

Characteristic CSF finding in Guillain-Barré syndrome?


Albuminocytologic dissociation (elevated CSF protein with a normal cell count)

Medication overuse headache


characterized by ? near-daily headache in the setting of regular use of acute headache
medications

Nervous system -DONE 3


have Pre-existing ? preexisting headache disorder (eg, migraine)

Dx reqmnt ? s/s for ≥3 months

Medication overuse headache


Common features ?

headache that is present upon awakening and brief symptom relief followed by rebound


pain

Most common meds causing medication overuse headache ?

butalbital-containing products, acetaminophen, and opioids

Management of medication overuse headache ?

cessation of the culprit drug with consideration of concomitant initiation of preventive


therapy.

How does chronic excessive noise exposure cause hearing loss ?


sensorineural hearing loss due to the irreversible death of hair cells in the cochlea

Military personnel are often exposed to loud noises, with pilots being especially at risk (due to
jet engine noise).

people serving in the US military (even if not in combat) are required to have regular hearing


evaluations.

Mechanism of damage in chronic excessive noise expo?

via both mechanical damage and metabolic overload of cochlear hair cells.

S/s of noise induced hearing loss ?

experience bilateral, high-frequency SNHL

what type of deafness is seen in otosclerosis ?


conductive hearing loss ( noise induced is SNHL)

asymmetric hearing loss ( symmetric in SNHL noise induced )

Pilots suffer from middle ear barotrauma . How ? Vs noise induced hearing
loss ?
Due to frequent, rapid changes in altitude

conductive or sensorineural, symmetric or asymmetric ( SNHL , symmetric )

ear pain /pressure + ear effusion on tymp memb retraction ( absent )

When does ossification of cochlea occur ?


follows from significant risk factors (eg, severe otosclerosis, meningitis, temporal bone
fracture)

Nervous system -DONE 4


not due to noise exposure

permanent high-frequency hearing loss

Does tympanic membrane fibrosis cause s/s ?


Asymptomatic, sequela of otitis media or barotrauma

why is it necessary to terminate a generalized convulsive status


epilepticus (GCSE) with BZD ?
bcz Prolonged seizures are associated with neuronal injury and death

brain that has seized for >5 minutes (status epilepticus) is at increased risk of developing
permanent injury due to excitatory cytotoxicity. > affecting the cortex

generalized convulsive status epilepticus (GCSE)


Following therapy pt with generalized convulsive status epilepticus (GCSE) regain
responsiveness in 10-20 minutes

D/D of persistent unresponsive state ?

1.Sedation due to persistent effects of benzodiazepines or

2.ongoing seizure activity without physical manifestations (ie, nonconvulsive status


epilepticus).

how to diff8 between the above two ?

electroencephalography

if seizure activity despite BZD and AED - continuous infusion of BZD

propofol or pentobarbital for refractory cases

Mx of persistent unresponsiveness due to sedation via BZD in pt


with generalized convulsive status epilepticus ?
>Flumazenil not used as increases risk of seizure

managed with supportive therapy (eg, securing airway and assisting breathing if necessary)

when a patient with Parkinsonism experiences orthostatic hypotension,


impotence, incontinence, or other autonomic symptoms - what is
suspected ?
multiple system atrophy (Shy-Drager syndrome) - a degenerative disease

C/o riley day syndrome ( familial dysautonomia ) which presents at birth

multiple system atrophy (Shy-Drager syndrome) Characterized by ? x3


1.Parkinsonism

Nervous system -DONE 5


2.Autonomic dysfunction- Autonomic s/s- postural hypotension, abnormal sweating,
disturbance of bowel or bladder control, abnormal salivation or lacrimation, impotence,
gastroparesis, etc)

3.Widespread neurological signs (cerebellar, pyramidal or lower motor neuron)

Mx of multiple system atrophy ? x3


Anti-Parkinsonism drugs - ineffective

intravascular volume expansion with fludrocortisone, salt supplementation, alpha-adrenergic


agonists

application of constrictive garments to the lower body

Idiopathic orthostatic hypotension is seen secondary to?


degeneration of postganglionic sympathetic neurons

Riley-Day syndrome or familial dysautonomia


Inheritance - autosomal-recessive

age group - children of Ashkenazi Jewish ancestry

presentation time - @birth

s/s - feeding problems and low muscle tone , dysfunction of ANS

severe orthostatic hypotension (swings from severe hypertension to postural


hypotension).

Mx of Cavernous sinus thrombosis ?


broad-spectrum intravenous antibiotics and prevention or reversal of cerebral herniation.

Dx - imaging modality of choice for cavernous sinus thrombosis ?

MRI with magnetic resonance venography

Red flag s/s that point to cavernous sinus thrombosis ?


1.severe headache

2.bilateral periorbital edema

3.cranial nerve III, IV, V, and VI deficits.

how does cavernous sinus thrombosis occur ?


because facial/opthalmic venous system is valveless

uncontrolled infection of the skin, sinuses, and orbit spreads through venous system

S/s of cavernous sinus thrombosis ?

1.intolerable Headache (M.C )

Nervous system -DONE 6


2. low-grade fever and periorbital edema(several days later )2/2 impaired venous flow in
the orbital veins

3.Vomiting + papilledema ( 2/2 rise in ICP)

4.Cranial nerves III, IV, V, and VI dysfunction

5.Unilateral symptoms can rapidly become bilateral - 2/2 communication between the


cavernous sinuses

Inflammation of the cavernous sinus subsequently results in life-threatening CST and


intracranial hypertension. s/s ( (eg, headache, binocular palsies, periorbital
edema, hypoesthesia, or hyperesthesia in V1/V2 distribution)

Cavernous sinus thrombosis vs Periorbital cellulitis ?


Both presents with fever and eyelid erythema/edema

inflammation extends beyond orbital septum eg cheeks ( confined to orbital septum )

intolerable headache ( unusual )

b/l face and eye involvment ( u/l usually)

opthalmoplegia common ( uncommon)

Pt age >50 with acute monocular vision loss , after 2 months of headache
and lateral scalp tenderness to palpation .
dx ? - giant cell arteritis (GCA;aka temporal arteritis)

MOST COMMON Opthalmic findings of Giant cell arteritis ?


Anterior ischemic optic neuropathy (AION)

Cause - partial or complete occlusion of the posterior ciliary artery

Fundoscopy? - pale, edematous disc with blurred margins.

Pt with parkinsonian features and recurrent falls , works independently


currently , Started on meds . , Now reports disturbing vision Dx ?
hallucinations -2/2 dopamine agonist (eg, pramipexole)

s/e less common with levodopa

Can this be due to dementia with lewy bodies ?

NO because pt doesnt have dementia in the first place.

diagnosis of dementia requires marked cognitive impairment that impairs activities of daily
living (eg, shopping, cooking).

Mx of hallucinations in Parkinsons disease.


trial to change to levo-dopa

Nervous system -DONE 7


if persists - try 2nd gen low potency antipsychotics (eg, quetiapine, pimavanserin).

Hallucinations can occur in parkinsons irrespective of meds so Mx complicated , Usually


hallucinations in parkinsons disease is 2/2 to t/t

most common psychiatric disturbance seen in Parkinson disease (PD)


Depression - hopelessness, suicidality , depressed mood and anhedonia (inability to
experience pleasure in previously enjoyed activities) - HELPS DIFF8 MDD FROM PD

Dx criteria for dementia with lewy bodies ?


patient with dementia meets 2 of the following 4 criteria:

1.visual hallucinations

2.parkinsonism

3.REM sleep behavior disorder

4.fluctuating cognitive function.

diagnosis of dementia requires marked cognitive impairment that impairs activities of


daily living (eg, shopping, cooking).

hallmark feature of frontotemporal dementia?


early change in personality (eg, disinhibition, apathy).

Frontotemporal dementiia other findings

compulsive behaviors (eg, hoarding, hyperorality).

executive dysfunction.

Memory deficits tend to appear later in the disease course

How is frontotemporal dementia different from other dementating diseases ?

FTD has an early age of onset (ie, 50s-60s)(only this is diff from others) and a strong
hereditary component

subclavian steal syndrome


due to severe atherosclerosis of the left subclavian artery proximal to the origin of the
vertebral artery.

Significant stenosis or occlusion leads to decreased pressure in the distal subclavian artery
and reversal ("steal") of blood flow in the ipsilateral vertebral artery.

Subclavian steal syndrome - which artery is more affected Rt vs Lt ?


left subclavian artery - likely due to sharper curvature and more turbulent blood flow (leading
to atherosclerosis).

Physical findings ?

Nervous system -DONE 8


lower brachial systolic blood pressure (eg, >15 mm Hg) in the affected arm

systolic bruit in the supraclavicular fossa on the affected side

S4 due to LVH 2/2 systemic HTN

S/s of Subclavian steal syndrome ?

ischemia in the affected upper extremity - esp when exercising that extremity

vertebrobasilar ischemia (eg, dizziness, ataxia, dysequilibrium)

Dx of subclavian steal syndrome?

Doppler ultrasound or magnetic resonance angiography.

Mx of subclavian steal syndrome?

lifestyle management (eg, lipid-lowering interventions, smoking cessation) and sometimes


stent placement

internal carotid artery stenosis vs Subclavian steal syndrome ?


transient symptoms of ocular or cerebral ischemia eg, amaurosis fugax, limb weakness (
Dizziness and dysequilibrium 2/2 to vertebrobasilar eg, cerebellar ischemia)

Vertebral artery stenosis vs Subclavian steal syndrome


Both present with dizziness or dysequilibrium 2/2 to vertebrobasilar (eg, cerebellar)
ischemia

arm ischemia or a blood pressure difference in the upper extremities ABSENT ( PRESENT )

Most common s/s of putaminal ie basal ganglia hemorrhage ?


almost always involves the adjacent internal capsule, leading to

1. contralateral hemiparesis and hemianesthesia due to disruption of the corticospinal


and somatosensory fibers in the posterior limb of internal capsule

2.conjugate gaze deviation toward the side - damage of frontal eye field efferents in the
anterior limb

most common cause of spontaneous deep intracerebral hemorrhage?


Hypertensive vasculopathy involving the small penetrating branches of the major cerebral
arteries

How does chronic HTN ppt intraparenchymal brain hemorrhage?


Chronic hypertension leads to the formation of Charcot-Bouchard aneurysms, which
may rupture and bleed within the deep brain structures

Most common locations of bleed in chronic HTN ?

1.basal ganglia (putamen ) - supplied by the lenticulostriate arteries - branches of MCA

Nervous system -DONE 9


2.cerebellar nuclei

3.thalamus, and pons

Crohns disease , Wt loss as h/o small bowel resections, Admitted for t/t of
C. Diff , since a day disorientation to time or place , unsteady gait and
abduction of rt eye limited Dx ?
Wernickes enceph - due to thiamine def 2/2 malnutrition ie wt loss -CD

Iatrogenic Wernickes encephalopathy?


precipitated in malnourished patients by the administration of dextrose

depletes the last remaining stores of thiamine, a vitamin cofactor for enzymes involved
in glucose metabolism.

Korsakoff syndrome is more common in Wernicke enceph in alcoholic abusers

KS occurs less common in non abusers of alcohol with WE

Non exertional heatstroke Vs exertional heat stroke ?


Cause ?

Extremes of age - unable to get fluids or remove themselves from hot


environment (hot/humid environments while performing extreme physical activity)

Anticholinergic toxicity vs heat stroke ?


temp - less than 104 ( severe hyperthermia ≥40 C ie 104 F)

s/s - pupillary dilation , flushing ( absent )

REM sleep behavior disorder (RBD) is characterized by?


dream enactment (ie violent /automatic motor movements during sleep) during REM sleep
atonia.

dream enactment because REM asso sleep atonia is absent / incomplete

When does REM sleep behaviour disorder occur ?

at least 90 minutes after sleep onset ( ie onset of REM sleep)

REM sleep behaviour disorder is more likely to occur in men age >50 years

RBD - strong association with the future development of alpha-


synuclein neurodegenerative disorders.

Alpha-synuclein is a synaptic protein that accumulates in some neurologic


conditions.

Pt with REM sleep behaviour disorder are prone to which neurological


condition ?

Nervous system -DONE 10


Parkinson disease (PD) or another disorder of alpha-synuclein
neurodegeneration (dementia with Lewy bodies or multiple system atrophy).

Women with migraine
particularly those associated with aura, are at increased risk of ischemic stroke.

Symptoms of aura may include

1.visual (eg, wavy lines, visual loss)

2.sensory (eg, paresthesia, numbness)

3.auditory (eg, sounds, hearing loss), and

4.motor (eg, tremors, weakness) dysfunction.

Medications contraindicated in migraine ?


Estrogen-containing contraceptives are contraindicated in patients with migraine with
aura due to the increased risk of ischemic stroke

estrogen-containing contraceptives (eg, combined estrogen/progestin pill, vaginal ring,


transdermal patch)

Indications for migraine prophylactic t/t ?


1.frequent or long-lasting episodes

2.disabling symptoms despite abortive treatment

3. no relief with abortive meds

4.Overuse abortive medication (eg, [NSAIDs]) and have rebound headache

Multiple sclerosis common symptoms ?


1.Optic neuritis: monocular visual loss, painful eye movements, and afferent pupillary defect

2.Transverse myelitis: motor and sensory loss below the level of the lesion with bowel and
bladder dysfunction. First flaccid paralysis (spinal shock) then spastic paralysis with
hyperreflexia.

3..Internuclear ophthalmoplegia: demyelination of the medial longitudinal fasciculus

4Cerebellar dysfunction: intention tremor, ataxia, and nystagmus

Which HIV pts are susceptible to neuropathy ?


greatest in older patients with long-standing HIV

high viral loads, low CD4 counts

history of exposure to neurotoxic antiretroviral medications (eg, didanosine, stavudine).

First line t/t to reduce sensory s/s in HIV neuropathy ?

Gabapentin

Nervous system -DONE 11


HIV peripheral neuropathy effect on motor strength ?
motor neurons are relatively spared, motor strength is usually intact.

Antiretroviral treatment reduces the risk of progression of HIV neuropathy.

syringomyelia
disorder in which a fluid-filled cavity (ie, syrinx) forms within the spinal cord.

Most common place ? - cervical or thoracic spine

syringomyelia most associated with Chiari type 1 malformation but may also occur with
spinal cord inflammation, infection, neoplasms, or trauma.

disturbance of the crossing spinothalamic tracts (STTs) in the anterior white commissure

Syringomyelia - s/s ?
loss of pain/temperature sensation in "cape" distribution

vibratory/proprioceptive sensation is typically intact as dorsal columns spared

Continued syrinx enlargment s/s ?


may affect lateral corticospinal tracts - UE area & anterior horn gray matter

weakness that disproportionately affects the upper extremities compared with the lower


extremities.

Spinal cord compression is a medical emergency?


YES , must be evaluated immediately with an MRI of the spine.

IV drug use and spinal cord compression s/s . Dx?


IVDU > epidural abscess (even without fever) > compression > stat MRI spine for dx

Lyme disease - subacute encephalopathy (meningitis/encephalitis) and


axonal polyneuropathy . Timing ?
occurs months to years after infection

GBS vs Spinal cord compression ?


NOT characterized by UMN signs ( present below the site of compression)

Nontraumatic subarachnoid hemorrhage


most commonly due to ruptured saccular (berry) aneurysm

SAH presents with = severe thunderclap headache (maximal intensity reached in <1 minute)

Pt with negative head CT scans and suspected SAH?

Nervous system -DONE 12


undergo lumbar puncture, which classically reveals elevated opening pressure and
xanthochromia

Non contrast CT -may not detect low-volume bleed

How to diff8 wether increased RBC is due to traumatic Lumbar puncture or


Subarachnoid hemorrhage ?
traumatic LP - RBCs/ blood clear with subsequent samples ( doesnt clear with subsequent
samples)

Subarachnoid hemorrhage - Lumbar puncture findings ?


>elevated opening pressure and increased RBCs

>xanthochromia (yellow CSF) due to RBC breakdown products ie bilirubin

HSV encephailitis vs Subarachnoid hemorrhage?


Both have elevated RBCs

LP findings - elevated WBC (normal)

systemic s/s - fever (not present)

Pt with unprovoked first seizure . NBSIM ?


(CT) scan of the brain without contrast is the initial imaging study of choice

Reason : - exclude acute neurologic problems due to structural causes (eg,


intracranial/SAH/mass) that might require urgent intervention

When is EEG done for unprovoked first seizure?

First CT brain without contrast - to rule out structural brain lesions

then EEG to stratify pt for need of maintenance anti epileptics

neuroimaging modality of choice in elective situations for epilepsy


structural causes ?
Magnetic resonance imaging as more sensitive than CT

Elective meaning chosen by the pt rather than urgent necessary

Multiple sclerosis vs Myaesthenia gravis


Both can cause weakness, blurred vision

symptoms typically relapse and remit slowly over days to weeks (  fatigable weakness that
fluctuates throughout the day)

Myaesthenia gravis - It is most common in?


women of childbearing age

Nervous system -DONE 13


pregnant and postpartum women are at increased risk for both disease onset and
exacerbation

When does myasthenia gravis affect men ?


later in life (ie, sixth to eighth decade).

incidence of MG less frequent than women - C/o MG more frequently in women and child
bearing age

Diabetic amyotrophy Causes ?


focal pain, weakness, and muscle wasting in the proximal legs

associated with weight loss

NO vision changes / fluctuating weakness , UE involvment is rare

Multiple sclerosis MRI findings ?


multifocal, ovoid hypo-/hyperintense lesions, white matter lesions in CNS

particularly the periventricular and subpial white matter of the cerebrum (corpus callosum)


and the optic nerves, brainstem, and spinal cord.

Multiple sclerosis . Mx ?
Acute multiple sclerosis exacerbation t/t ? - high-dose intravenous glucocorticoids. - as this
hastens neurologic recovery.

MS flare who are refractory to corticosteroid therapy : - Plasma exchange ie plasmapheresis

for long-term disease suppression of multiple sclerosis/ Disease modifying agents used
for chronic maintenance therapy: - immunomodulators (eg, interferon beta, natalizumab,
glatiramer)

Outcome : decrease the frequency of exacerbations in patients with relapsing-remitting or


secondary, progressive forms of MS

Interferon beta and glatiramer acetate NO ROLE IN ACUTE EXACERBATION

sinusitis vs Intracranial hypertension


both present can present with frontal headache that worsens with leaning forward

assosciated with nasal congestion and purulent nasal discharge ( ( neurologic s/s eg blurred
vision and falls )

Cushing reflex ?
(hypertension, bradycardia, respiratory depression) is a worrisome finding suggestive of
brainstem compression.

S/s of intracranial hypertension ?

Nervous system -DONE 14


1.headache (worse at night)

2.nausea/vomiting

3.mental status changes(eg, decreased level of consciousness, cognitive dysfunction)

4.symptoms worsen with maneuvers that increase ICP (eg, leaning forward, Valsalva, cough)

On examination - intracranial hypertension may present with ? - papilledema and focal


neurological deficits

Initial interventions for all patients with carotid artery stenosis should
include ?
1.intensive medical management (ie, aspirin, statin, blood pressure control) and

2.counseling on lifestyle changes (eg, exercise, smoking cessation).

symptomatic patients (ie, TIA or ischemic stroke in the distribution of the


affected vessel within the previous 6 months) with high-grade carotid
stenosis (70%-99%) . NBSIM ?
carotid endarterectomy to reduce future stroke risk

Pt with carotid artery 100% occlusion , persistently disabling neurologic deficits or life
expectancy <5 years - UNLIKELY TO BENEFIT

first-line therapy for those undergoing a carotid endarterectomy.?


aspirin

Clopidogrel used if cannot tolerate aspirin.

Characteristics of vascular dementia ?


1.focal neurologic findings (eg, right-sided hemiparesis, sensory deficits)

2.risk factors for stroke (eg, age, hypertension)

3.early mild forgetfulness

4.sudden change ( eg unprovoked fall )

5.Stepwise decline ( eg worsening / additional s/s following initial s/s - another fall and mood
problems )

Neuroimaging finding in vascular dementia ?

cortical and/or subcortical infarction with or without deep white matter changes from
chronic ischemia

Vascular dementia vs Fronto-temporal dementia ?


stepwise decline and neurologic deficits ( such a pattern not seen )

Vascular dementia vs Alzheimers disease ?

Nervous system -DONE 15


abrupt and stepwise course ( slowly progressive disease )

focal neurological findings eg weakness/loss of sensation ( language deficits- not speech, and
spatial disorientation )

neuroimaging finding of Alzheimers Disease


medial temporal lobe atrophy is characteristic of Alzheimers Disease

Fronto-temporal dementia vs Alzheimers disease ?


Behavioral changes often precede memory impairments (in contrast to AD).

most common cause of dementia?


Alzheimer disease, the most common cause of dementia, is characterized by insidious
memory loss followed by behavioral changes.

Psychiatric s/s in alzheimers disease ?

Psychotic features (delusions, hallucinations) - middle or late course of disease

esp delusions with paranoid themes

Alzheimers disease initial s/s


1.memory loss for recent events (eg, poor short-term memory, naming current president)

2.executive dysfunction

3.impaired visuospatial skills

Alzheimers disease Later s/s


1. language difficulties

2.apraxia, and

3.behavioral changes (eg, apathy, disinhibition, suspiciousness)

Pseudodementia vs Alzheimers disease ?


h/o of depression + prominent mood s/s ( ABSENT )

Pseudodementia (also referred to as dementia syndrome of depression)

Asterixis common causes ? x3


Uremic encephalopathy (due to acute or chronic renal failure)

hepatic encephalopathy (HE)

CO2 retention

Asterixis a bilateral, nonrhythmic, alternate flexion and extension movement at the wrist
(flapping) - WHEN wrist is extended with arms outstretched

Nervous system -DONE 16


diagnosis of uremia is based on?
clinical symptoms/signs and not on an absolute BUN level.

Reduced renal clearance of uremic toxins leads to high levels of blood urea
nitrogen (BUN) and symptoms of lethargy and somnolence

Uremic encephalopathy is an indication for?


urgent hemodialysis

Symptoms of uremia typically appear at a BUN level of >100 mg/dL but can develop at lower
levels

Prophylactic meds for cluster headaches


verapamil or lithium

Abortive meds for acute attack of cluster headaches ?


100% oxygen - most rapid-acting and effective , no s/e or

subcutaneous sumatriptan - contraindications (eg, ischemic cardiac disease, pregnancy)

Trigeminal neuralgia vs Cluster headaches ?


1.Pain -repetitive, unilateral, shooting/shock-like pain in the distribution of CN V ( stabbing pain
- retro-orbital area )

2.Triggered by - chewing, talking, touching certain parts of the face ( occurs at sleep )

3. duration - lasts seconds to minutes ( 15 min -90 min)

4. duration - lasts seconds to minutes ( 15 min -90 min)

Bells palsy - classic presentation - no secondary s/s . Next step for Dx ?


no further diagnostic workup

other causes of bells palsy r/o with physical examination and h/o

Does bells palsy present with a prodrome ?


may be preceded by a prodrome of auricular pain or dysacusis (ie, distortion of sound).

Pain in spinal cord compression vs degenerative joint disease ?


Pain is usually worse in the recumbent position( improves with recumbency. )

Pain - worse - recumbent position (due to distension of the epidural venous plexus
when lying down)

Brain ischemic strokes vs spinal cord compression ?


Pain - absent ( present , usually back pain)

Neurological findings - focal unilateral neurologic deficits ( usually symmetric and B/ L )

Nervous system -DONE 17


Progress - acute deficits over hours ( over days )

GBS vs Spinal cord compression ?


Reflexes - diminished or absent but not hyperactive reflexes(late - hyperactive)

Diabetic polyneuropathy vs spinal cord compression


Back pain - absent ( present usually)

DTR - diminised /absent ( hyper-reflexic )

Common fibular neuropathy is typically the result of?


leg immobilization, leg crossing, or protracted squatting.

Manifestations are usually transient ie lasting hours ( C/o Multiple Sclerosis s/s lasts days
to weeks and include foot drop and sensory changes over the dorsal foot and lateral shin)

Physical examination

impaired ankle dorsiflexion and great toe extension with preserved plantar flexion and
reflexes.

Spinal cord compression - malignancy related


A history of progressive back pain is common (particularly with malignancy) and often 6-8
weeks before neurologic manifestations.

Effect of central retinal artery occlusion on vision ?


>monocular painless acute vision loss

MCC ? - embolized atherosclerotic plaque from the ipsilateral carotid artery.

How to differentiate between ischemic and hemorrhagic stroke?


noncontrast CT scan of the head

absence of hemorrhage suggests ischemic stroke

Pt with ischemic stroke - not eligible for thrombolytic therapy or


mechanical thrombectomy NBSIM ? Rationale ?
antiplatelet agents (eg, aspirin, clopidogrel)

decrease the risk of early recurrent ischemic stroke.

given even before the full workup (eg, ECG, carotid duplex ultrasonography) for a


stroke source is complete

If pt with ischemic stroke - is undergoing thrombolysis or mechanical


thrombectomy - Should antiplatelets be given ?

Nervous system -DONE 18


NO , antiplatelet therapy is typically withheld for 24 hours to decrease the risk of bleeding
(eg, hemorrhagic conversion)

Ischemic stroke due to Afib . Give parenteral anticoagulation?


No , should not be used in acute mx of ischemic stroke - >increases the risk of
hemorrhagic conversion

Oral anticoagulation (eg, rivaroxaban, apixaban, warfarin) - used for long-term stroke
prevention when stroke is caused by a cardioembolic event

Which Vitamin Def does Metformin cause and after how much time ?
vitamin B12 deficiency after ≥5 years of treatment

due to metformin altering calcium homeostasis in the gastrointestinal tract, thereby impairing


calcium-dependent vitamin B12 absorption in the terminal ileum.

Pt on metformin -
neuropsychiatric manifestations (eg, irritability, crying spells), >sensory ataxia (swaying when
standing with eyes closed)
and >positive Babinski sign? MCV , Hb normal
Dx ?

Vit B12 def due to metformin

lack of hemat abnorm . does NOT exclude dx of B12 def

Does Vitamin B12 def. Always present with hemat. Abnorms ?


NO , sometimes just presents with neuropsychiatric manifestations

lack of hemat abnorm. DOES NOT exclude b12 def

look out for mild thrombocyotpenia leukopenia

Neurological abnormalities in Vitamin B12 deficiency. due to myelin


DAMAGE.S/S ?
1.Dorsal columns > impaired vibratory sensation/proprioception and sensory ataxia (gait
impairment that worsens when the eyes are closed [positive Romberg sign])

2.Lateral corticospinal tracts, leading to positive Babinski sign

3.Myelinated peripheral nerves, leading to lower extremity paresthesias

4.Myelinated fibers in the brain, leading to neuropsychiatric manifestations (eg, irritation,


mood changes)

Does diabetic polyneuropathy cause babinsky sign ?


NO , it is a peripheral neuropathy , positive babinski wont be seen

Nervous system -DONE 19


Hypothyroidism neurological findings ?
peripheral polyneuropathy- lower extremity paresthesia and sensory loss in a stocking-
glove distribution

DTR may be diminished

Babinski sign - NOT seen

Profound hemineglect is usually due to a lesion impacting?


nondominant parietal lobe, which is responsible for spatial organization.

majority of patients, particularly right-handed patients, the right parietal lobe is


the nondominant lobe

How does neglect manifest in pt with nondominant(rt ) parietal lobe


damage?
1.Sensory neglect

>includes tactile, visual, and even auditory neglect

>assessed with extinction testing (bilateral sensory stimulus experienced only on one
side).

2.Motor-intentional neglect

decreased spontaneous movement on the neglected side despite normal strength and use
of the "incorrect" (ie, normal) arm when instructed to move the affected arm.

3.Conceptual neglect

include a lack of awareness or concern about the deficits. In severe cases, patients may
even say that half of their body does not belong to them.

Screening test for pt with damage to nondominant parietal lobe ?


Drawing a clock which requires both motor and sensory components

putting numbers only on the right side - indicates damage

Damage affects motor , sensory and conceptual neglect

chronic inflammatory demyelinating polyneuropathy (CIDP) VS Lambert -


eaton Myasthenic syndrome ?
muscle involvment - Proximal + distal muscle weakness ( proximal muscle weakness with
hyporeflexia)

sensory defects present ( absent )

chronic inflammatory demyelinating polyneuropathy (CIDP) VS Multiple


sclerosis ?
often symmetric ( plaques in CNS so rarely symmetric )

Nervous system -DONE 20


LMN signs present ( absent )

Chronic inflammatory demyelinating polyneuropathy VS Vitamin B12 def


Both can present with loss of vibration and position sense.

DTR s ? LMN signs - hyporeflexia (spastic paresis with hyperreflexia)

most common presenting sign of Parkinsons Disease ?


asymmetric rest tremor in the distal part of an upper extremity.

3 cardinal signs of Parkinson disease?


rest tremor, rigidity, and bradykinesia

The presence of at least 2 of these 3 on physical examination is grounds for a clinical


diagnosis of this disease.

Best tool to confirm dx in parkinson disease ?


Physical examination - 2 out 3 cardinal signs confirms the dx

>Lumbar puncture is usually normal in PD

>Striatal dopamine transporter imaging (DaTscan) helps distinguish parkinsonian syndromes


from essential tremor

Physical examination findings that lead to Dx of parkinsons disease ?


1.Tremor:

A resting 4 to 6 Hz tremor with a "pill rolling" quality

Frequently first manifests in one hand> slowly generalize to involve the other side of the
body and the lower extremities

2.Rigidity:

Baseline increased resistance to passive movement about a joint which may be uniform
(lead pipe) or oscillating (cogwheel)

3.Bradykinesia:

Difficulty initiating movements, as when starting to walk or rising from a chair

Narrow-based, shuffling gait with short strides and without arm swing (festinating gait)

Micrographia (small handwriting)

Hypomimia (decreased facial expression)

Hypophonia (soft speech)

4.Postural instability:

Flexed axial posture

Loss of balance during turning or stopping

Nervous system -DONE 21


Loss of balance when pushed slightly from a stationary bipedal stance

Frequent falls

Brown-sequard syndrome Findings ?


1.Ipsilateral hemiparesis: lateral corticospinal tract - at the level of the cord injury and below

2.Ipsilateral diminished proprioception, vibratory sensation, and light touch: dorsal


columns > at the level of the cord injury and below

3.Contralateral diminished pain and temperature: lateral spinothalamic tract >occurs 1-2


levels distal to the cord injury and below because LST sensory neurons send axon
projections up 1-2 levels prior to decussating (crossing over)

Brown sequard syndrome in cervical region ?


Above s/s + ipsilateral Horner syndrome (ptosis, miosis, anhidrosis)

Brown-sequard syndrome > reflexes ?


Initially ipsilateral reflexes absent & babinski absent due to spinal shock

later as spinal shock resolves - ipsilateral hyper-reflexia and positive babinski

Vascular dementia - strategic infarction frontal lobe VS alzheimers


Both can cause memory loss and personality changes

Abrupt decline in functioning eg months after stroke ( AD -  insidious onset of memory loss
over months to years followed by behaviour changes later stage of disease)

rapid development of memory loss, executive dysfunction, and behavioral


changes 4 months following a stroke?
most consistent with vascular dementia (VaD)

VaD, the second-most common form of dementia after Alzheimer disease

Red flags indicated that central vertigo in pt req urgent evaluation for
cerebellar stroke or hemorrhage ?
1.prominent stroke risk factors (eg, hyperlipidemia, hypertension, diabetes mellitus).

2.new-onset headache.

3.neurologic signs/symptoms (eg, weakness/numbness of the face or limbs, dysarthria)

CT preferred over MRI - as completed faster , IF CT negative go with MRI

Can Thiazide use cause vertigo ?


Hypotension and hyponatremia > lightheadedness or presyncope

DOESNT cause VERTIGO

Nervous system -DONE 22


Pt with Acute CENTRAL vertigo . Can you treat with Meclizine ? NBSIM ?
NO , used for peripheral vertigo

Urgent Head imaging CT - to eval . stroke / hemorrhage

The first-line treatment for cognitive symptoms of Alzheimers Disease?


cholinesterase inhibitor

may improve quality of life and cognitive function (eg, memory, language, thought, reasoning)

not been shown to alter the disease course in any type of dementia.

Alzheimers disease T/t ?


Mild to Moderate Alzheimers Disease > Donepezil, galantamine, and rivastigmine

Moderate to Severe Alzheimers Disease > Memantine, an NMDA receptor antagonist

Memantine blocks action of glutamate on NMDA receptor

Alzheimers disease behavioural s/s Vs depression?


AD present with years of cognitive decline eg memory loss followed by mood /personality
changes in later course( mood changes + cognitive impairment concurrent )

female - planning to be pregnant . Has seizure h/o - t/t with phenytoin since
5 years . NBSIM ?
Phenytoin - should be slowly tapered and discont. as rapid withdrawal may result in
seizure recurrence.

Can you switch to valproic acid ?

NO because , increased risk of congenital anomalies such as neural tube defects (eg,
spina bifida) and dysmorphic facial features

Effect of phenytoin on pregnancy ?


fetal hydantoin syndrome (eg, orofacial clefts, microcephaly, nail/digit hypoplasia, cardiac
defects, dysmorphic facial features)

Cause of urinary incontinence in pt with anterior cerebral artery Stroke ?


damage to the cortical micturition centers of the mesial frontal lobe

Does descending aortic dissection ( type B ) cause vertebral artery


dissection ?
NO, vert. art. dissection is caused by Type A , aortic dissection

How does vertebral artery dissection present ? eg due to type A aortic


dissection?

Nervous system -DONE 23


neck pain, ischemic stroke, and headache

may cause posterior spinal cord ischemia > loss of proprioception/vibration sensation below
lesion

Effect of Type B descending aortic dissection on the spinal cord ?


Anterior cord syndrome due to thoracic spinal cord ischemia > interruption of the
intercostal and/or lumbar arteries (off the aorta) that feed the anterior spinal cord

Risk of anterior spinal cord ischemia is greatest at the T10-T12 levels, where blood flow
is lowest.

Effect on spinal cord?


Type A aortic dissection : Posterior spinal cord ischemia

Type B aortic dissection : anterior spinal cord ischemia

most common cause of lobar intracranial hemorrhage (ICH)


cerebral amyloid angiopathy (CAA)

most often occurs in patients age >75

abnormal beta-pleated sheet protein (amyloid) infiltrates cerebral blood vessels, increasing
fragility and leading to rupture with resultant spontaneous ICH

intracranial hemorrhage (ICH) in cerebral amyloid angiopathy vs


Hypertensive hemorrhage ?
Mc in occipital and parietal lobes, usually sparing the ventricles and deep brain structures (
involves deep brain structures )

Restless Leg Syndrome?


characterized by dysesthesia (eg, crawling and itching feelings) and an irresistible urge to
move the legs when at rest

Transverse myelitis ?
immune-mediated disorder

characterized by the infiltration of inflammatory cells into a segment of the spinal cord,

leading to neuron and oligodendrocyte cell death and demyelination.

assosciated with multiple sclerosis and systemic disease (eg, sarcoidosis)

Transverse myelitis - Inflammation localizes to ≥1 contiguous spinal cord segments

Causes rapidly progressive myelopathy characterized by:

motor weakness that progresses from flaccid to spastic paraparesis with UMNS.

Nervous system -DONE 24


autonomic dysfunction including bowel/bladder incontinence or retention and sexual
dysfunction.

sensory dysfunction including pain, paresthesia, or numbness with a distinct sensory


level (ie, lowest spinal cord level with intact sensation)

Cauda equina syndrome vs Transverse myelitis ?


associated with severe back pain and sensory loss that involves the thighs and
buttocks ( distinct sensory loss and no back pain )

Effect of sulfonamides eg TMP-SMX on the eye ?


occasionally trigger Angle Closure Glaucoma due to swelling of structures (eg, lens, retina,
choroid) in the posterior chamber.

Cluster headache vs Angle closure glaucoma


Both causes - acute periorbital pain, conjunctival injection, and lacrimation

Progression recurrent , short-lived minutes to hours attacks ( not episodic )

Asso with autonomic s/s eg miosis, u/l ( no autonomic s/s, fixed mid dilated pupil )

nausea/vomiting- absent ( present )

Argyll robertson pupill?


miotic and irregular and characterized by normal pupillary constriction with accommodation but
not with light.

Asso with Late neurosyphilis , ie prostitutes pupill - accommodates but doesnot react

Gullain Barre syndrome vs Epidural abscess


both present with motor and sensory s/s

MRI - normal ( Epidural fluid collection with rim enhancement)

s/s - no fever , no spine tenderness(fever and severe pain /tender spine)

h/o - GI/ upper resp infection ( IVDU or distant infection)

Vertebral Mets MRI ?


extradural bony lesions with cord compression

Trigeminal neuralgia usually presents UNILATERALLY , When does it


present bilaterally ?
Multiple sclerosis

Trigeminal neuralgia is characterized by recurrent and sudden-onset severe, stabbing pain


along the V2 (maxillary) and V3 (mandibular) branches of the trigeminal nerve

Nervous system -DONE 25


B/L trigeminal neuralgia.
Can it be due to herpes zoster ?

NO , because, Herpes virus reactivation usually occurs in V1 ( trigem. neuralgia V2 V3),


involves 1 dermatome at a time , and ITS Unilateral

Basilar migraines Cause ?


cerebral vasospasm involving the brainstem

How does basilar migraines present ?

basilar aura symptoms (eg, vertigo, dysarthria, tinnitus, diplopia) without motor weakness

followed by a migraine-type headache

Central venous catheter use ?


pts experiencing shock and/or receiving vasopressor therapy

delivery of vasopressors (and other medications or intravenous fluids)

indwelling arterial catheters use ?


continuous blood pressure monitoring and frequent arterial blood sampling

Common complications for brachial artery catheterization ( insert and


removal) ?
median nerve injury- lies medialy to brachial artery

local compression due to hematoma/swelling(common)- sensory disturbances - median nerve


distribution

laceration(rare)- motor weakness + sensory disturbance

lacerated by the seeker needle used to insert the arterial catheter (rare), compressed by a
postprocedural hematoma or local swelling (common)

Brachial artery cannulization - thrombosis . Outcome?


Acute limb ischemia - 6Ps - distally due to minimal collateral circulation

Internal jugular vein cannulation - neurological complication ?


Brachial plexus injury (eg, compression from hematoma)

nerve root (ie, dermatomal), trunk, or cord distribution

U/L Medial longitudinal fasciculus lesion cause ?


lacunar stroke in the pontine artery distribution

B/L Medial longitudinal fasciculus lesion cause ?multiple sclerosis.

U/L Medial longitudinal fasciculus lesion presentation ?

Nervous system -DONE 26


affected eye (ipsilateral to the lesion) is unable to adduct and the contralateral eye abducts
with nystagmus

Patients with amaurosis fugax and concomitant carotid artery disease ?


have an increased risk of stroke.

Define myoclonus ?
sudden, involuntary muscle contraction or relaxation that results in movement of limbs or
joints

posthypoxic myoclonus (PHM), a form of secondary myoclonus that commonly occurs after
cardiac arrest

Posthypoxic myoclonus 2 forms


1.Myoclonus status epilepticus (MSE)

>Acute form

>develops within 24 hours after the initial hypoxic insult while the patient is still
unconscious.

characterized by a generalized (often symmetric) myoclonus that typically involves the


axial, limb, and facial muscles; intermittent eye opening, upward gaze deviation, and
swallowing movements are also common.

Persistent MSE is considered a marker of poor prognosis.

2. Lance-Adams syndrome

>chronic form of PHM

presents days to weeks after the initial insult once the patient has regained
consciousness.

It is typically focal in nature , exacerbated by action

negative (relaxation) myoclonus also occurs, leading patients to drop objects or fall.

Mx of Acute / chronic posthypoxic myoclonus ?


administration of antiepileptic agents (eg, clonazepam, levetiracetam) and supportive care

Fasciculations vs Myoclonus ?
visible twitching of muscles without movement of joints. ( involves movement of joints )

Facial palsy due to Sarcoidosis vs Parotid gland tumour ?


acute onset ( slow , gradual growing )

hypercalcemia ( not present )

B/L parotid enlargement ( U/L painless mass )

Nervous system -DONE 27


Hilar LAD sometime A/S ( cervical LAD )

Myaesthenia Gravis epidemio


presents in the sixth to eighth decade in men and

the second and third decade in women

How does the icepack test work in Myaesthenia gravis ?


ice pack applied over eyelids for several minutes

cold temp -inhibs the breakdown of Ach at the NMJ > improves muscle strength

Pt with positive icepack test > NBSIM > confirm with Ach receptor antibodies

most common sites for hypertensive hemorrhage in descending order


include the?
1.basal ganglia (putamen)

2.cerebellar nuclei

3.thalamus

4.pons, and cerebral cortex.

When Is epidural hematoma an emergency condition ? NBSIM ?


Pt with focal deficits / signs of intracranial HTN

emergent neurosurgical hematoma evacuation because treatment delay may lead to


death 2/2 brain herniation.

small epidural hematoma without significant neurologic dysfunction. Mx ?

Close observation and serial (every 6-8 hours) CT scans

How does strychnine poisoning present ?


blocks inhibitory (glycine) neurotransmission within the spinal cord

powerful involuntary - muscle contractions - episodic - tonic clonic activity - in a fully awake
patient

Strychnine, an ingredient in rodenticide and some illicit drugs

how does scorpion sting present ?


cause neuronal membrane hyperexcitability - uncontrolled, repetitive firing of axons

fasciculations and jerking of the extremities in awake pt

intense pain at the site of bite

Functional ( psychogenic ) tremor vs Physiologic tremor ?


distraction - improves with dist. ( no change)

Nervous system -DONE 28


frequency- low frequency high amp ( high freq , low amp)

Effect of alcoholic cerebellar degeneration on reflexes ?


Muscle hypotonia may present > pendular knee reflex > persistent swinging of limb ie more
than 4 swings after eliciting a Deep tendon reflex is abnormal

Does alcoholic cerebellar degeneration present with tremor?

Later stages > postural tremor of fingers , thighs

Lead poisoning
Neuropsychiatric manifestations

Sensorimotor neuropathy (eg, weakness with dorsiflexion), short-term memory loss

Hematologic manifestation

Microcytic anemia (often with basophilic stippling on peripheral


smear)hyperuricemia (due to impaired purine metabolism)

Inhibition of enzymes responsible for heme and RNA synthesis in both bone marrow and
mature erythrocytes can lead to microcytic anemia with basophilic stippling

Chronic lead toxicity can cause?


hypertension, nephropathy, and more pronounced neuropsychiatric symptoms (eg, psychosis).

Lead is absorbed predominantly via the lungs in adults - stored predominantly in - skeleton -
released slowly - effect over decades

treatment for symptomatic lead poisoning


chelation therapy with an agent such as calcium disodium EDTA.

Why is it imp to separate pt from source of lead poisoning before T/t ?

because ,if exposure contd , chelation therapy increases lead absorption from source

Aneurysm cranial nerve compression


Oculomotor nerve (CN III) palsy : aneurysm of the adjacent posterior communicating
artery.

optic nerve (CN II) : - aneurysm of internal carotid or anterior communicating artery

trochlear (CN IV) or abducens (CN VI) nerve palsy : aneurysm affecting the superior
cerebellar or anterior inferior cerebellar artery, respectively.

patient with sudden-onset, severe headache and a suggestive CN palsy.


NBSIM ?
head CT scan with angiography to evaluate for active SAH or an enlarging aneurysm with
impending rupture.

Nervous system -DONE 29


Cluster headache vs Subarachnoid hemorrhage - aneurysm ?
Headache - U/L ( B/L constant )

ANS - ipsilateral parasympathetic hyperactivity( signs due to CN compression)

Pupils - constriction ( dilation if CN -3 involved )

Cerebral toxoplasmosis vs Primary CNS lymphoma


Both have ring enhancing lesions - MRI

multiple ring-enhancing lesions ( solitary, irregular, nonhomogenous ring-enhancing lesion)

CSF - EBV DNA absent ( present - 100% specificity )

Bacterial brain abscess vs Primary CNS lymphoma in AIDS with IVDU


both have ring enhancing lesion

s/s of primary infection in other area present ( viral reactivation )

EBV DNA in CSF

AIDS dementia complex - brain imaging ?


cortical and subcortical atrophy and ventricular enlargement

HIV-associated dementia occurs almost exclusively in?


untreated HIV-infected patients with advanced disease (eg, CD4+ <200)

What is cervical spondylosis?


a degenerative condition associated with spinal canal narrowing

due to the formation of osteophytes in the lateral vertebral bodies and ossification of the


posterior longitudinal ligament/ligamentum flavum.

Cervical spondylosis is most common in which age group ?

older adults

Lumbar spinal stenosis is most commonly caused by?


degenerative joint disease.

In DJD, disc herniation and facet osteophytes impinge upon the spinal cord.

Bells palsy upper face s/s ?


inability to close the eye, weakness raising the eyebrow

Acute stroke vs bells palsy

upper face spared ( upper face involved )

New Neurological findings in a pt with Multiple sclerosis

Nervous system -DONE 30


Transient ischemic attack vs Acute multiple sclerosis attack?

neuro deficits transient <24 hours ( lasts days to weeks )

Transient Ischemic attack . Mx ?

modifying identifiable risk factors eg quit tobacco use

initiating aspirin

starting a statin for hyperlipidemia

reducing blood pressure.

35 y woman - h/o of crohns t/t with intestinal surgery , 4 weeks of


worsening fatigue and painful parethesia in feet, Normal DTRs, MCV 98, Hb
8 g/dl, Dx?
vitamin B12 deficiency

Can it be heavy metal poisoning ?

no because

it presents within 24 hours ( 4 weeks )

motor neuropathy + weakneess and areflexia

which neurological s/s presents initially in Vitamin B12 deficiency ?


myelinated nerves in PNS - affected first > lower extremity paresthesias with no other
neurologic findings.

look out for mild thrombocytopenia and mild leukopenia

Bell palsy . Pathophysiology ?


reactivation of neurotrophic viruses (eg, herpes simplex virus), resulting
in inflammation, edema, and degeneration of the myelin sheath. of CN VII

How do you identify delirium with Confusion Assessment Method (CAM) ?


assesses 4 fundamental features of delirium:

1.acute change (eg, hours to days) and fluctuating course (eg, intermittent)

2.inattention (eg, easy distractibility)

3.disorganized thinking (eg, confusion)

4.altered consciousness (eg, daytime somnolence)

delirium diagnosis formally requires both features 1 and 2 plus either feature 3 or 4

What does delirium indicate ?


acute brain failure and should be considered a medical emergency

Nervous system -DONE 31


Effect of delirium on dementia ?
accelerates the progression of dementia

even in pts without h/o of dementia - single episode of delirium increases risk of cognitive
decline

Due to neuroinflammation, decreased cerebral blood flow, impaired neurotransmitter function

Delirium - characterized by ?
acute-onset, fluctuating consciousness, most frequently seen in elderly hospitalized
patients

Does delirium resolve with t/t ?

It usually resolves with treatment.

pulse oximetery in Carbon monoxide poisoning?


Pulse oximetry is usually normal, because the oximeter cannot differentiate between
oxyhemoglobin and carboxyhemoglobin

Acid base balance in CO poisoning ?


anion gap metabolic acidosis (AGMA) due to lactic acidosis from peripheral tissue hypoxia

CO disrupts oxidative phosphorylation in mitochondria.

binds to Hb - reduction in O2carrying capacity.

left shifft in Hb - decreasing O2 unloading in the tissues.

Loss of consciousness and temperature ?


Severe hypothermia (ie, body temperature <28 C) is typically required to cause loss of
consciousness.

Bradycardia and a reduced respiratory rate are expected with such a degree of hypothermia.

Management of aminoglycoside ototoxicity ?


immediate discontinuation of the medication to prevent further damage.

How does aminoglycoside ototoxicity present ? x3


>B/L symmetric /asymmetric hearing loss

Imbalance and sensation of objects moving (oscillopsia) - b/l vestibular systems affect (
true vertigo - u/l vestibular system involvment)

Positive head thrust test

>damage hair cells in the cochlea (causing hearing loss) and/or the vestibular system (causing
imbalance)

Nervous system -DONE 32


positive head thrust test (ie, inability to maintain visual fixation during forced, rapid head
movement). This test evaluates the vestibuloocular reflex, which is affected by peripheral but
not central vestibulopathies

Utility of head thrust test


evaluates the vestibuloocular reflex, which is affected by peripheral but not central
vestibulopathies

positive head thrust test (ie, inability to maintain visual fixation during forced, rapid head
movement) - indicating peripheral vestibulopathy

fundoscopy in optic neuritis ?


normal as inflammation occurs behind the optic nerve head.

CT findings in Diffuse axonal injury ?


Cannot be detected on CT , since DAI - is microscopic injury to axons

MRI - more sensitive

MRI > minute punctate hemorrhages in the white matter and blurring of the gray-white
interface.

Most common cause of death in Amyotrophic lateral sclerosis ?


respiratory failure

How does ALS cause respiratory failure ? x3


1.Inspiratory muscle (eg, diaphragm) atrophy and weakness, > poor inspiratory strength

2Expiratory muscle weakness> ineffective cough and poor bronchial clearance

3.Bulbar muscle weakness >dysphagia and chronic aspiration

first-line therapy for ALS patients with respiratory insufficiency?

Noninvasive positive-pressure ventilation (NIPPV)

improves respiratory function by opening the upper airway and providing positive end-
expiratory pressure to improve atelectasis.

tetanus is common in which population ?


unvaccinated or inadequately immunized

When does Tetanus present after inoculation ?

as toxin takes time to transport in retrograde way > few days to several weeks following
inoculation

Tetanus presents with fever, muscle stiffness, trismus/lockjaw (inability to open his mouth
completely), and painful muscle spasms

Nervous system -DONE 33


Also sweating, dysphagia, labile blood pressure, and tachycardia.

How does tick paralysis present ?


paresthesias and fatigue followed by rapidly progressive ascending paralysis (which may be
asymmetrical)

absence of fever and sensory abnormalities

normal CSF examination

Mx of tick paralysis
skin examination for tick removal

improvement within hours of tick removal

Pathophysio of tick paralysis ?


neurotoxin release

the tick needs to feed for 4-7 days for the release of neurotoxin

GBS vs Tick paralysis ?


Both have ascending paralysis

onset of paralysis - over days to weeks ( over hours )

autonomic dysfunction - present in 70% ( absent )

CSF ex- albuminocytologic dissosciation ( normal )

GBS can be difficult to diff8 from tick paralysis , however, meticulous search for a tick is very
easy to perform. If a tick is found, extensive workup and unnecessary treatment can be
avoided.

Pt with myaesthenia gravis ,on outpatient pyridostigmin , has crisis ,


intubated. Continue pyridostigmine ?
Following intubation, Achesterase inhibitors (eg, pyridostigmine) > are temporarily
held to reduce excess airway secretions and the risk of aspiration

What if corticosteroids are ineffective in myaesthenia crisis ?

Other immunomodulatory therapy (eg, mycophenolate mofetil or azathioprine) can be


considered > takes weeks to take effect

hospitalized pt with delirium - agitated . t/t with BZD ?


No because paradoxical effect in the elderly by causing disinhibition, thereby worsening
delirium.

How does delirium vary throughout the day ?

Worse at night , improves in the day

Nervous system -DONE 34


Pt with migraine headches . Now has daily b/l frontal headaches with
nausea and blurry vision .NBSIM ?
MRI brain , as new onset diff headache is a warning sign

Which warning signs should be assessed in pt with evaluation of


headache ?
1.Neurologic findings: Seizure, changes in consciousness, specific deficits

2.Differences compared to prior headaches: Change in frequency, intensity, characteristics

3.Other: New at age >40, sudden onset, trauma, present on awakening

What should be done if a warning s/s is present in pt with prior headache ?

MRI of the brain to determine the cause.

elderly pt - 2months headache and nausea worse in morning , resistance


to passive flexion in Left UE and LE , unsteady gait , u/l weakness , Dx ?
Brain tumour - headache and nausea indicate rise in ICP

Can this be lacunar infarction ?

no because, it doesnt present with headaches and nausea for months

presents with U/L weakness

Can it be due to tabes dorsalis /Vit B12 def ?

No because , both dont present with U/L weakness and neither presents with headache
and nausea

How does brain tumours present ?


most common symptom is a dull headache associated with >1 of the following

1.Nausea and vomiting (due to increased ICP)

2.Focal neurologic manifestations (due to tumor invasion or compression)

3.Symptoms worsening during the night or with positions that raise ICP (eg, bending,
coughing)

Evaluation for brain tumour ?

MRI , fundoscopy to check for raised ICP

Brain tumors are often clinically silent until they grow large enough to compress neurologic
structures or raise intracranial pressure (ICP)

Cerebral lobes and speech involvement ?


Nondominant temporal lobe

inability to comprehend emotional gestures (sensory aprosodia)

Nervous system -DONE 35


Nondominant frontal lobe

inability to convey emotion through speech (motor aprosodia)

no expressive aphasia

Dominant temporal lobe lesions - caudal superior temporal gyrus

wernickes area

unable to - comprehension (receptive aphasia) , ability to speak nouns (anomic aphasia),


repetition (conductive aphasia)- arcuate fasciculus involvement

Dominant frontal lobe - posterior inferior frontal gyrus - brocas area

comprehend and follow commands but are unable to verbalize or write properly
(expressive aphasia)

functions as brain to motor function of larynx and mouth

Prognosis of postconcussive syndrome ?


most patients will improve within 3 months following Traumatic Brain Injury

s/s may persist for >6 months.

Myaesthenia gravis pt undergoing thymectomy. complication and


prevention ?
transient worsening of symptoms (eg, myasthenic crisis with respiratory failure)

Prevention : - perioperative pyridostigmine and immunosuppressants

Cause of headache in ischemia of cerebral circulation eg posterior


circulation ?
due to trigeminal innervation of much of the cerebral arterial vasculature

Jaw jerk reflex - indicates ?


mediated by the trigeminal nerve (CN V)

bilateral upper motor neuron lesions. eg ALS

Bulbar s/s in Amyotrophic lateral sclerosis ?


1.Dysphagia:

Tongue and pharyngeal muscle weakness> poor control of liquids.

Palatal weakness > fluid refluxing into the nasopharynx and out the nose.

2.Dysarthria:

Lip and tongue weakness > slurred speech

palate weakness > hypernasal speech.

3.Jaw jerk reflex - due to B/L UMN lesion

Nervous system -DONE 36


4.Tongue atrophy and fasciculations:

LMN signs are due to denervation of tongue musculature.

MRI - Alzheimers disease


early stages - normal

later stages - temporal lobe atrophy most prominent in the hippocampi and surrounding


medial temporal lobes

MRI - frontotemporal dementia


Atrophy of the frontal lobes

remember FTD earlier age of onset than other dementias

MRI - primary progressive aphasia


Atrophy of the parietal lobes

primarily affects language early in the disease.

Brain abscess - headache character ?


severe, unilateral, and resistant to analgesics

Is fever always present in brain abscess ?

No , presents in only 50% of cases

Brain abscess - Rhizopus/nocardia vs strep/staph ?


CD4 <100, poorly controlled DM , immunosuppresants ( healthy pt with other infection )

most common cause of Wernicke aphasia?


infarct in the - inferior division of middle cerebral artery

S/s

caudal superior temporal gyrus > wernickes aphasia

lateral parietal and temporal heteromodal association cortex affected > unable to integrate
function from various sensory inputs - eg difficulty using daily object - holding upside down

inferior optic radiations > right upper visual field deficit

Most common cause of brocas aphasia ?


stroke of superior division of middle cerebral artery

Brocas Aphasia vs Wernickes aphasia ?


Motor involvment : - Contralateral hemiparesis due to primary motor cortex involvment (
spared )

Nervous system -DONE 37


Pt with GCSE ( Generalized convulsive status epilepticus) - seizure
terminated with IV lorazepam . NBSIM ?
nonbenzodiazepine antiepileptic medication should be administered to prevent seizure
recurrence eg fosphenytoin, phenytoin, levetiracetam, or valproic acid.

Once GCSE is stabilized . NBSIM ?

neuroimaging (eg, brain MRI, head CT) to evaluate underlying cause

Any patient who does not return to a normal state of consciousness after medical therapy
should undergo continuous electroencephalography to rule out nonconvulsive status
epilepticus

Pt on warfarin . Used over the counter meds for cold s/s . Now has
intracerebral hemorrhage . How?
Acetaminophen >potentiates anticoagulant effect of warfarin

decongestants > increased BP

Mx of warfarin -associated intracerebral hemorrhage?


Initially - IV Vitamin K - takes effect in 12-24 hr

Prothrombin complex concentrate (PCC) - rapid reversal of warfarin for short term

Mx reduce the risk of death and permanent disability.

Why lumbar puncture shouldnt be done in acute intracerebral


hemorrhage?
risk of inducing herniation as a result of increased intracranial pressure

How does cervical radiculopathy occur ?


common in older pts

physical activity or trauma > stress on neck > disc herniation / nerve root compression from
underlying cervical spondylosis. - Cervical spondylosis is marked by cervical spine
degeneration

How does oculomotor palsy present ?


pupilary dilation due to paralysis of sphinter pupillae

ptosis due to paralysis of the levator palpebrae superioris

downward and outward eye due to unopposed LR and SO action

CN III has parasympathetic outside and motor fibers inside

How to differentiate etiology of oculomotor nerve palsy ?


Non–pupil-sparing CN III palsies

Nervous system -DONE 38


mass effect usually due to an intracranial aneurysm until proved otherwise

Dx - CT angiography / MR of the head immediately

Pupil-sparing CN III palsies

caused by microvascular ischemia and are associated with DM, HTN, hyperlipidemia,


and advanced age.

CN III has , parasympathetic outside - not prone to ischemia , motor fibers inside - prone


to ischemia

Treatment of Tremor - dominant Parkinsons Disease


younger patients - trihexyphenidyl.

older patients - amantadine - avoid anticholinergic effect of trihex

Tremor in PD - resting tremor of 5-7 Hz that is asymmetric and associated with rigidity.

Lead poisoning vs Acute intermittent porphyria


Both have abdominal pain and neuropsychiatric manifestations and peripheral neuropathy

ANS - not involved ( dysautonomia - tachycardia, diaphoresis, tremor)

Urine findings - absent ( positive urobilinogen / porphyrins)

lab - anemia, normal sodium ( no anemia, , SIADH > +/- hyponatremia )

Constitutional - fatigue and joint pain ( absent )

hypokalemia ECG findings ?


1.broad flat T waves

2.U waves

3. ST depression, and premature ventricular beats

Arrythmias due to Hypokalemia ?


1.Atrial fibrillation

2.torsades de pointes, and 3.ventricular fibrillation

causes of hypokalemia ?
K wasting diuretic eg thiazides

diarrhea , vomiting

anorexia, and hyperaldosteronism

Hypokalemia - s/s
weakness, fatigue, and muscle cramps

Severe hypokalemia (serum concentration <2.5 mEq/L) S/S ?

Nervous system -DONE 39


1.Flaccid paralysis & hyporeflexia

2.tetany

3.rhabdomyolysis

4.arrhythmias

Normal pressure hydrocephalus - mechanism ?


decreased cerebrospinal fluid (CSF) absorption by the arachnoid granulations

Idiopathic normal pressure hydrocephalus assosciated with ?

chronic periventricular ischemia and increased venous resistance, which may alter normal
arachnoid function.

Lumbar puncture in NPH?

normal CSF studies and opening pressure

clinical improvement with large removal of CSF - diagnostic

Normal pressure hydrocephalus VS Alzheimers disease ?


cognitive deficits and gait impairment - occur together ( cognitive deficits early and gait
impairement later )

Urinary urgency - assosciated with NPH early course- later incontinence ( not asso )

Hyperreflexia - often present ( absent )

CT - ventriculomegaly with no sulcul enlargement ( ventriculomegaly + sulcal enlargement due


to brain atrophy)

Acute toxicity - phenytoin - S/S ?


1.signs of cerebellar dysfunction

horizontal nystagmus (ie, nystagmus on lateral gaze)

ataxia (eg, wide-based gait)

dysmetria (eg, on finger-nose testing)

2.slurred speech, and nausea/vomiting are common

3.hyperreflexia ( may occur)

Phenytoin - inhibits voltage-gated sodium channels

Severe toxicity - phenytoin ? S/s ?


altered mental status (eg, lethargy, confusion)

coma & death

paradoxical seizures

Phenytoin - inhibits voltage-gated sodium channels

Nervous system -DONE 40


Rapid IV infusions of phenytoin / fosphenytoin S/E ?
hypotension and bradyarrhythmia.

Phenytoin - inhibits voltage-gated sodium channels

How does phenytoin toxicity ppt ?


after dosing adjustments

cytochrome p450 system inhibitors

phenytoin is highly protein-bound and is hepatically metabolized by the cytochrome p450


system

Mx of Phenytoin toxicity ?
supportive care with gastric decontamination and possibly hemodialysis

Acute/chronic subdural hematoma vs Phenytoin toxicity ?


acute - coma at injury or soon ( AMS presents later )

chronic - cognitive impairement , somnolent over days ( cerebellar dysfunction presents before
confusion / AMS)

Nystagmus - not found ( present )

chronic venous insufficiency vs Restless leg syndrome ?


both have discomfort in legs

Pain : - pain increases when walking and improves when lying down ( pain improves with
walking , worsens when resting )

Presbycusis
common in the elderly and presents with bilateral, symmetric, sensorineural hearing loss.

What is thalamic pain syndrome / Dejerine-Roussy syndrome ?when does


it present ?
weeks to months after lacunar stroke of thalamus esp posterolateral thalamus

severe paroxysmal burning pain over the stroke affected area that is exacerbated by light
touch(allodynia)

Mid brain stroke presentation ?


ipsilateral oculomotor nerve palsy

ataxia (due to damage of the superior cerebellar peduncle), and >contralateral hemiparesis
(cerebral peduncle).

VPL and VPM stroke thalamus ?

Nervous system -DONE 41


sudden-onset contralateral sensory loss involving all sensory modalities (ie, pure sensory
stroke) - face and body

S/s other than characteristic of thalamus stroke ?

transient hemiparesis, athetosis, or ballistic movements due to disruption of neighboring


basal ganglia & corticospinal fibers of posterior limb internal capsule

ventral posterolateral and ventral posteromedial nuclei of the thalamus transmit sensory
information from the contralateral side of the body and face, respectively

Vertebral body squaring is an early sign of?


ankylosing spondylitis

No radiculopathy in ankylosing spondylitis

How does atlantoaxial subluxation present ? vs cervical spondylosis


compressive cervical myelopathy ( radiculopathy )

neck pain that radiates to the occiput ( episodic pain radiating to dermatome )

progressive spastic paresis, Lhermitte sign -shock like sensation down the spine-, and LE
manifestations ( single myotome/dermatome involved)

Does odontoid process fracture cause radiculopathy ?


spinal cord compression (myelopathy), not radiculopathy.

setting of high-impact trauma (eg, motor vehicle collision)

Pt drug addict - left leg fracture . not resolving with ketorolac (NSAIDs) .
would you give morphine ?
Yes , with close follow-up care to avoid relapse

Acute Pain management ?

Irrespective of substance abuse h/o

first give NSAIDS eg ketorolac , if pain persists IV morphine

Pain managment scenarios


long-term management of pain in patients who require daily, continuous pain relief

Transdermal fentanyl

mild or moderate pain who are unable to take oral or rectal preparations

Intravenous acetaminophen

CSF finding in tuberculous meningitis


elevated adenosine deaminase

lymphocytosis

Nervous system -DONE 42


moderate increase WBCs

mildly elevated protein

low glucose

Tuberculous meningitis course ?


M. TB replicates slowly

weeks of progressive headache, nausea/vomiting, fever, neck stiffness, and malaise.

Herpes encephalitis course ?


acute (<1 week) fever, headache, focal neurologic deficits, confusion, and/or seizures are
present

Meningococcal meningitis course ?


progresses within hours (not weeks) to septic shock and death

Memory loss - Alzheimers disease vs MDD ?


less concerned about their memory loss , but shows increased effort to compensate eg
incorrect answers /confabulate ( concerned about cognitive decline but poor effort)

Does MDD in elderly present with depressed mood ?


demonstrate cognitive impairment and slowing rather than depressed mood

Antithrombotic therapy in
Bioprosthetic valves > only aspirin

Mechanical prosthetic valves > anticoagulation

PVT can still occur IRRESPECTIVE OF THERAPY

initial diagnostic workup of a first-time seizure in an adult should include ?


1.basic blood tests (eg, serum electrolytes, glucose, calcium, magnesium, CBC , renal and
liver function tests)

2. toxicology screen to evaluate for metabolic and toxic causes

Once metabolic and toxic causes of first time seizure r/o NBSIM ?

neuroimaging and EEG.

Stroke dementia
Large artery infarction (often causing an overt stroke) produces a cortical-type VaD

s/s - related to involved specific cortical region

course : - Stepwise worsening

Nervous system -DONE 43


small arterial infarction - lead to a subcortical-type VaD

s/s - focal motor deficits (eg, reflex asymmetry), abnormal gait, urinary symptoms, and
psychiatric symptoms (eg, depressive syndromes)

Course : - gradual declining course.

When does Parkinson disease dementia present?


parkinsonism predates dementia symptoms by ≥1 year

Dementia with Lewy bodies distinguished from Parkinson disease dementia primarily by
timeline

Does frontotemporal dementia cause focal neurological deficits ?


not associated with focal neurologic deficits

Apart from GI and upper resp infection . what can cause GBS ?
acute HIV infection can also trigger GBS

onset of GBS with regards to HIV ? 


most commonly occurs prior to the onset of AIDS (ie, while there is still a robust immune
response) and can be the initial HIV presentation

Stroke patients with severe hyperglycemia (ie, blood glucose >180 mg/dL)


MX ?
treated with insulin because hyperglycemia may worsen brain injury

Always After thrombolytics

Advise to decrease risk in pt with family h/o of alzheimers disease ?


Aggressive treatment of cardiovascular risk factors (eg, hypertension, diabetes,
obesity/physical inactivity), especially in mid-life, can help reduce risk.

pts with family history of Alzheimer disease are at increased risk of developing the disease

Pharmacotherapy of Dementia with Lewy Bodies


carbidopa-levodopa for parkinsonism and cholinesterase inhibitors for cognitive impairment.

Mx For psychotic s/s in dementia with lewy bodies ?


a low-dose second-generation antipsychotic

avoid first gen antipsychotics > due to the severe neuroleptic sensitivity(severe parkinsonism
and impaired consciousness with neuroleptic administration) in DLB

Acute polyneuropathy vs chronic polyneuropathy ?


Chronic polyneuropathy - due to systemic disease (eg, diabetes mellitus, HIV infection)

Nervous system -DONE 44


Acute polyneuropathy - due to toxins or medications

compared to chronic - acute toxic polyneuropathy are more likely to have significant pain
(burning) accompanying the sensory loss

Common medications implicated in polyneuropathy ?


metronidazole

dapsone,

fluoroquinolones

amiodarone

digoxin

cessation of the offending medication is often the first step in management of med induced
polyneuropathy

Giant cell arteritis vs Temporo Mandibular Joint disorder


Both present with headache and joint pain

Age - >50 ( less than 50 )

S/s - fever , fatigue , weight loss ( absent )

Trigeminal neuralgia pt treated with oxcarbazepine is at risk for what ?


risk for hyponatremia due to increased sensitivity to antidiuretic hormone.

Why is oxcarbazepine preferred over carbamazepine for trigeminal neuralgia ?

because OX better tolerated (eg, less nausea/vomiting, less risk for leukopenia)

herpes zoster neuralgia vs Trigeminal neuralgia


pain paroxysms = can occur at night ( doesnt occur at night )

H/O = h/o of rash 1 week before or after neuralgia ( no rash h/o )

Pathogenesis viral reactivation (nerve compression )

How does chronic anticholinergic toxicity present ?


absent s/s of acute tox ie tachycardia, hyperthermia, flushed dry skin, urinary retention

subtle, nonspecific symptoms (eg, confusion, altered mental status, dysarthria

Removal of the anticholinergic medication leads to resolution of symptoms.

Myasthenic crisis - respiratory failure is often preceded by ?


by increasing generalized or bulbar muscle weakness

Wernicke area, located in the?

Nervous system -DONE 45


caudal superior temporal gyrus of the dominant temporal lobe

How does wernickes aphasia present ?

well-articulated nonsensical speech

lack of language comprehension.

lack insight into their condition

In a right-handed patient, the dominant hemisphere is nearly always the left.

Does nondominant lobe lesions affect speech ?


do not impact speech because hemispheric dominance is defined by the side that most
controls speech

Why is there partial horners syndrome with internal carotid artery


dissection ?
ANHIDROSIS does not occur as the sympathetic fibers responsible for facial
diaphoresis travel along the external carotid artery

partial Horner syndrome characterized by ptosis and miosis ( NO anhidrosis )

Why is partial horners present in carotid artery dissection ?


Distension of the sympathetic fibers that travel along the internal carotid artery lead to partial
Horner syndrome

When is plasmapheresis done in pt with transverse myelitis ?


for patients who have significant motor symptoms or do not respond to glucocorticoids

Transverse myelitis prognosis post t/t ?

Most patients partially recover, but persistent deficits are common.

Lacunar infarct of internal capsule vs Subdural hematoma ?


Both can present with contralateral hemiparesis

ABSCENCE OF impaired consciousness or cortical signs eg, aphasia, neglect,somnolence ,


confusion ( PRESENT )

Toxic metabolic encephalopathy vs Subdural hematoma ?


Focal neurological deficits - ABSENT ( PRESENT )

What is todd paralysis ?


Transient limb weakness following partial seizure activity is known as postictal (Todd)
paralysis.

Pathophysiology of trigeminal neuralgia ?

Nervous system -DONE 46


compression of the trigeminal nerve root as it enters the pons, leading to demyelination
and atrophy of the nerve.

most common causes

1.due to a vascular loop

2.neoplastic growth

3.multiple sclerosis plaque.

TB meningitis vs bacterial meningitis ? Course ?


slowly progressive symptoms over weeks ( severe symptoms within hours/days)

Myotonic Dystrophy is characterized by?


myotonia (delayed muscle relaxation after contraction) with progressive muscle atrophy and
weakness.

Findings of classic Myotonic Dystrophy reflect the involvement of different


muscle groups
1.Involvement of the muscles of the face can lead to ptosis

2.flat affect (likely due to prolonged weakness)

3.lid lag after closing the eyes tightly

4.Oropharyngeal muscle weakness (eg, dysarthria) may also occur.

5.Involvement of the distal musculature (eg, intrinsic muscles of the hands, ankle


dorsiflexors) explains the classic grip myotonia (inability to quickly release the hand grip).

In which age group does myotonic dystrophy present and how ?


late adolescence or early adulthood with facial and distal muscle weakness and
myotonia (delayed muscle relaxation after contraction)

what is hemiballismus ? damage of what does this ?


unilateral, violent arm flinging caused by damage to the contralateral subthalamic nucleus.

Hemiballismus is very disruptive but tends to be self-limited.

Torticollis?
common form of focal dystonia involving the sternocleidomastoid muscle

occur idiopathically but is very often medication-related

Discontinuation of the causative agent(s) may improve symptoms.

What is athetosis ?
slow, writhing movements that typically affect the hands and feet

Nervous system -DONE 47


Athetoid movements are characteristic of Huntington's disease

What is chorea ?
brief, irregular, unintentional muscle contractions , non repetitive non rhythmic

movements tend to flow from one to another

Major risk factors for lacunar stroke ? x4


1.Hypertension

2.hyperlipidemia

3.diabetes

4.smoking

PRESENTATION - Lacunar infarcts commonly occur in the internal capsule lead to pure


motor hemiparesis due to injury of the corticospinal (posterior limb) and corticobulbar (genu)
tracts.

CT scan findings in lacunar stroke ?

Due to their small size, lacunes are often not appreciated on noncontrast CT

Brainstem strokes characteristically lead to?


ipsilateral cranial nerve deficits and contralateral deficits of the body (ie, crossed signs).

How does mid brain stroke present ?


ataxia due to dysfunction of the red nucleus

ipsilateral oculomotor (CN III) palsy

contralateral hemiparesis (corticospinal tract)

NO loss of pain and temperature sensation and NO Horner syndrome like lateral medullary
syndromes

What happens to HR in brain death ?


In brain death, vagal activity is already lost due to absent brainstem and cranial nerve
function

HR high , no effect on HR change when atropin admin.

Herpes enchephalitis
EEG findings in Herpes encephalitis ?

prominent intermittent high amplitude slow waves

Gold standard for Dx of herpes encephalitis ?

Polymerase chain reaction analysis of HSV DNA in CSF (highly sensitive and specific)

Nervous system -DONE 48


What kind of behavioural changes are seen in herpes encephalitis ?

hypomania, Klüver-Bucy syndrome (hyperphagia, hypersexuality), and amnesia

Parkinson disease, dementia with Lewy bodies, and multiple system


atrophy.
Shows degeneration of which brain structure ? - substantia nigra

hallmark neuropathological finding in Huntingtons Disease that can be


seen on neuroimaging
atrophy of the caudate nucleus and putamen (ie, neostriatum).

Once Myaesthenia Gravis is estd . NBSIM ?


chest imaging (eg, CT scan, MRI) to evaluate for thymoma and for possible surgical planning
ie thymectomy

Benefit of thymectomy in Myaesthenia gravis pts ?

associated with long-term clinical improvement in both patients with and without


thymoma.

MG confirmatory testing with acetylcholine receptor antibodies

Brain Mets Mx
Single brain metastasis in surgically accessible location, good performance ?

surgical resection followed by stereotactic radiosurgery (SRS) or whole brain radiation


therapy (WBRT) to the tumor bed

Multiple brain mets + poor performance status ?

Whole Brain Radiation Therapy or supportive care

Brachytherapy use in brain mets ?

in conjunction with surgery, or after recurrence following WBRT or surgery

Can chemotherapy be used in brain mets due to non squamous cell Lung carcinoma ?

NO , because NSCLC is not chemosensitive

used in chemo sensitive cancers such as small cell lung cancer, lymphoma,
choriocarcinoma

Lambert-Eaton myasthenic syndrome (LEMS) Symptomatic therapy . Mx ?


guanidine or 3,4-diaminopyridine - increase presynaptic acetylcholine levels

LEMS - refractory symptoms Mx ?

immunologic therapy with intravenous immunoglobulin or oral immunosuppressants (eg,


corticosteroids, azathioprine).

Nervous system -DONE 49


LEMS Dx ?

autoantibodies against voltage-gated calcium channels

electrophysiological studies

Lambert-Eaton myasthenic syndrome is frequently associated with an underlying


malignancy (eg, small cell lung cancer).

Essential tremor + Asthma on inhalers. Mx ?


Primidone

propranolol avoided in asthmatics and pts with heart block

Pt with RA +TNF inhib t/t, holosystolic murmur at Apex, subacute non-sp


s/s, subconjunctival hemorrhage, Dx ?
left-sided infective endocarditis (IE) with septic emboli to the eye and brain.

NBSIM ?

1.3 sets of blood cultures from 3 different venipuncture sites

2.transthoracic echocardiography (TTE) to evaluate for valvular vegetations or abscess

3.If TTE negative - TEE done

toxoplasmic encephalitis vs brain abscess due to septic emboli from Lt


sided endocarditis
NO because , it presents with

multiple, bilateral ring-enhancing lesions ( Multiloculated , rim enhancing fluid lesions)

Murmur absent ( present )

subconjunctival hemorrhae absent ( present )

what is hemiplegic migraine ?


familial disorder

migraine headache associated with unilateral motor deficits

migraine headache associated with unilateral motor deficits

How does subacute hydrocephalus present in pt with SAH ? vs


vasospasm ?
multiple neurologic deficits due to compression of the cranial nerves, brainstem, and/or
cerebral cortex(focal unilateral deficit)

Idiopathic intracranial hypertension


BMI = >30 kg/m2 -

Headache - holocranial headache

Nervous system -DONE 50


Vision changes - blurry vision, diplopia

pulsatile tinnitus - whooshing sound in the ears

CVS - hypertension and bradycardia via cushing reflex

pathology involves impaired absorption of CSF by the arachnoid villi

Most significant complication of idiopathic intracranial hypertension (pseudotumor cerebri)?

Blindness

Preferred imaging modality for idiopathic intracranial hypertension ?

MRI, often with venography / CT if emergency as faster

Cytology in lumbar puncture of IIH ?

Normal cell counts

Medications that can cause Idiopathic intracranial hypertension ?


glucocorticoids or vitamin A

Oral contraceptive pills - assosciated with it

Mx of Idiopathic intracranial hypertension?


weight reduction and acetazolamide (if weight reduction fails) - inhibits choroid plexus
carbonic anhydrase, thereby decreasing CSF production and IH

Mx of idiopathic intracranial HTN - when medical measures fail or visual


field defects are progressive ?
lumboperitoneal shunting or optic nerve sheath fenestration - helps prevent blindness

What is malignant hemispheric infarction ?


when an ischemic stroke causes massive cerebral edema and/or hemorrhagic
transformation

How does malignant hemispheric infarction (MHI) present ?

initially present with severe deficits > neurologic deterioration in the first 48 hours or more
gradual up to a week

Pt with suspected Malignant hemispheric infarction. NBSIM ?

noncontrast CT scan of the head -  emergently to determine the extent of edema


and/or hemorrhage

Can Malignant hemispheric infarction be managed with osmotic diuretics and


observation ?

although it helps reduce ICP , observation is not done after that

Decompressive hemicraniectomy is often necessary after imaging is done

Nervous system -DONE 51


Massive Ischemic stroke 2 outcomes ?
Cerebral edema - endothelial dysfunction and breakdown of the BBB > mass effect & inc ICP
> herniation

Hemorrhagic transformation -blood extravasates from injured cerebral vessels into the brain
parenchyma > larger the infarct > greater  risk of hemorrhagic transformation.

Radiculopathies( compression of the nerve root) due to ?Compare onset


disk herniation = acute symptom onset

progressive spinal spondylosis = subacute or progressive symptom onset

Cervical radiculopathy occurs most commonly in middle-aged men and may develop after
repetitive exercise (eg, golf)

Cervical radiculopathy vs thoracic outlet syndrome .


Hand placed over head / shoulder abduction test - improvement of radicular symptoms as
reduced tension on impinged nerve root ( worsens compressive symptoms )

shoulder abduction relief test is both diagnostic and therapeutic for short-term pain relief - in
cervical radiculopathy

Risk factors assosciated with multiple sclerosis ( other than table ) ?


Epstein-Barr virus infection

reduced sunlight exposure

Elevated intracranial pressure - effect on eye movement ?


In approximately 25% of cases, elevated ICP compresses the 6th (abducens nerve) cranial
nerve , leading to lateral gaze palsy and diplopia.

how does trauma cause spinal epidural abscess ?


development of a hematoma that may expand and become infected.

How does spinal epidural abscess cause sore throat or radiating pain ?
extension of the abscess into the retropharyngeal space

Delirium with agitation - elderly pts with dementia? Mx?


low-dose haloperidol or

Atypical antipsychotics (quetiapine, risperidone)

How long should the t/t be contd ? Why ?

shortest possible duration, as prolonged use of antipsychotics can increase mortality in the
elderly.

Nervous system -DONE 52


Avoid antipsychotics in pts with delirium and dementia with lewy bodies due to neuroleptic
hypersensitivity in these pts

Delirium with agitation - young patients - Mx ?


Lorazepam / other BZDs, avoided in elderly as metabolize slowly and worsen delirium

Avoid antipsychotics in pts with delirium and dementia with lewy bodies due to neuroleptic
hypersensitivity in these pts

How does Miller Fisher syndrome present ?


Variant of Guillain-Barré syndrome - same pathophy

ophthalmoplegia, ataxia, and areflexia ,strength is often preserved

Anti-GQ1b antibodies are highly sensitive for MFS

How does MFS cause opthalmoplegia?


high levels of GQ1b in CN III, IV, and VI - and MFS has high Anti-GQ1b antibodies

CSF = GBS albuminocytologic dissociation (ie, elevated protein, normal white blood cell count)

Mx =  plasmapheresis and intravenous immunoglobulin.

Fibromyalgia
Initial t/t of choice - Amitriptyline

If doesnt respond ? - Pregabalin, duloxetine, or milnacipran

Tool to dx fibromyalgia ?

widespread pain index and symptom severity scale

hallmark of prolonged seizures and can lead to persistent neurologic


deficits and recurrent seizures.?
Cortical laminar necrosis

Persistent seizure activity > excitatory cytotoxicity affecting the cortex

MRI - evidence of cortical hyperintensity on diffusion-weighted imaging suggesting infarction

What is blepharospasm ?
focal dystonia characterized by recurrent forceful contraction of the eyelid muscles

Trigger of spasm? Bright lights

Terminates spasm? touching or brushing the skin around the eye

Mx of blepharospasm ?

Mild cases - trigger avoidance eg dark glasses to block bright lights

Significant S/s - botulinum toxin injection

Nervous system -DONE 53


Is blepharospasm U/L ?

No, bilateral and symmetric

Blepharospasm vs eyelid twitching


spasm - eyelids closed - affects the vision (fleeting, minor lid contractions - donot affect vision)

b/l ( u/l)

Does wilsons disease have anemia ?


Yes , Coombs-negative hemolytic anemia due to red blood cell copper accumulation

Wilsons disease in young children vs young adults ?


liver disease - A/s LFT elevation ,hepatomegaly or fulminant liver failure ( liver disease
+ neuropsychiatric disease)

Food borne botulism vs Myaesthenia gravis


Pupils - sluggishly reactive , mydriatic ( pupils spared )

ANS dysfunction - present ( less prominent )

Symptom progression - rapid within 36 hrs ( less rapid )

most common presentation of ALS


Asymmetric limb weakness

bulbar symptoms (eg, dysarthria, dysphagia) are the second most common.

Suspected Giant cell arteritis . Mx ? Biopsy first or t/t first ?


to reduce the risk of blindness, T/t with systemic glucocorticoids should not be
delayed while awaiting biopsy results.

Biopsy findings remain detectable up to 30 days after the start of treatment.

Vision loss character in idiopathic intracranial HTN ?


transient vision loss lasting a few seconds with changes in head position

BPPV
due to crystalline debris (canaliths) in the semicircular canals that disrupt the normal flow of
fluid in the vestibular system.

leads to contradictory signaling from the corresponding canals on each side, which is
interpreted as a spinning sensation (vertigo)

Maneuver to Dx

BPPV - Dix-hallpike maneuver

Maneuver to reposition

Nervous system -DONE 54


BPPV - epley maneuver

How does riluzole help in amyotrophic lateral sclerosis ?


Riluzole glutamate inhibitor - reducing glutamate-mediated excitotoxicity

Which drug is added along with riluzole for ALS ?


edaravone, an antioxidant, which likely slows disease progression - esp early in disease
course

first-line medical treatment for idiopathic intracranial hypertension


Acetazolamide +/- furosemide

Pt with Diabetic neuropathy , urinary incontinence . Now has memory loss


and disorientation + dementia since one month . NBSIM ?
adverse effect of polypharmacy , possibly amitriptyline / oxybutinine , anticholinergic meds >
cognitive impairment

Elderly are susceptible to cognitive impairment due to which drugs ?


anticholinergic and sedating medications

Meds cause medication-induced syndrome that mimics dementia

Greatest risk factor for TIA and Acute ischemic stroke ?


hypertension

Risk factor management should prioritize lowering blood pressure because even mild


reductions can diminish stroke risk.

Patients with atrial fibrillation PLUS existing structural heart disease have an increased risk


of cardioembolic strokes.

Does risk of stroke due to smoking decrease after cessation ?


risk normalizes within approximately 4-5 years of cessation.

Does risk of stroke due to DM decrease after glycemic control ?


does not provide a significant reduction in risk.

Amyotrophic lateral sclerosis


leads to degeneration of cells in the anterior horn of the spinal cord

What does widespread fibrillations and positive sharp waves on electrophysiologic studies


reflect ?

reflecting spontaneous depolarization - denervated fibers upregulate acetylcholine


receptors > hypersensitive to acetylcholine and spontaneously discharge

Nervous system -DONE 55


What does pronator drift indicate ?
sensitive and specific for upper motor neuron or pyramidal/corticospinal tract disease

How is pronator drift performed ?

outstretch the arms with the palms up and eyes closed (so that only proprioception is


used to maintain arm position)

Pronator drift mechanism ?

UMN lesions cause more weakness in the supinator muscles compared to the pronator
muscles of the upper limb > affected arm drifts downward and the palm turns (pronates)
toward the floor.

Proprioception tests
passively moving the distal phalange of a digit up and down and having patients identify
direction of movement with eyes closed

romberg test

What kind of drift occurs in cerebellar dysfunction ?

upward drift (rather than the downward drift seen in pyramidal tract lesions) due to
hypotonia

cauda equina syndrome (CES), a medical emergency?


marked by compression of ≥2 spinal nerve roots in the lumbar cistern

cauda equina carries nerve roots from L2-L5, S1-S5 and the coccygeal nerve

Cauda equina syndrome vs Conus medullaris syndrome/lumbar spinal


cord involvment ?
both can cause perianal anesthesia and bowel/bladder dysfunction

Lower extremitis lower motor neuron signs eg, loss of reflexes ( UMN signs )

Cauda-equina syndrome characteristic ?


progressive lower back radicular pain (into one or both legs) plus ≥1 of the following

1.Motor deficits , eg reflexes absent

2.Patchy sensory loss -  saddle anesthesia - particulary suggestive

3.Rectal sphincter, bladder, and/or sexual dysfunction due to damage to S3-S5

saddle anesthesia (impaired sensation in the buttocks, perineum, and/or perianal area)

Pt with s/s suggestive of cauda equina syndrome ? NBSIM ?


urgent MRI of the lumbosacral spine to evaluate for CES

Surgical decompression within 24-48 hours - to prevent irreversible neurologic sequelae

Nervous system -DONE 56


Mcc of cauda equina syndrome ?

large, central lumbar disc herniation - MCc

epidural tumor or abscess or inflammation

Injury to ulnar nerve at wrist


Cause - hamate fracture, compression from a bicycle handlebar

numbness and paresthesia - medial side of hand

intrinsic hand weakness- described as clumsiness

Injury to ulnar nerve at elbow


cause : - prolonged leaning on the elbow, sleeping with the elbow in extreme flexion, direct
blow (ie, "hitting the funny bone") , ESRD - positioning during hemodialysis

Above s/s + decreased grip strength (due to denervation of the flexor muscles in the
forearm)

weaker hand flexion at the wrist (flexor carpi ulnaris).

Median nerve injury at wrist


sensory symptoms in the lateral fingers

Median nerve injury at elbow


sensory symptoms - Lateral fingers + sensory findings over the thenar eminence

Radial nerve injury at elbow


weakness of hand/finger extensor muscles and sensory loss over the posterior
forearm/dorsolateral hand

Radial nerve injury at wrist


lateral hand + motor deficits are typically minor.

Steppage gait is most commonly caused by?


L5 radiculopathy or neuropathy of the common peroneal nerve

L5 radiculopathy vs Pernoneal neuropathy ?


Back pain - present + radiating to foot ( absent )

foot eversion and dorsiflexion - weakness ( NO weakness , paresthesias and sensory loss
over the dorsum of the foot)

diagnosis can be clarified with electromyography and nerve conduction studies.

Thrombotic stroke vs embolic stroke ?

Nervous system -DONE 57


"stuttering" course with periods of worsening and improvement( symptoms maximal at onset)

Does cerebellar hemorrhage present with hemiparesis ?

NO

Pt with alzheimers disease on quietapine . Has pneumonia. cause ?


Atypical antipsychotic medications (eg, quetiapine)  -  increase the risk of CAP in elderly
patients - due to anticholinergic and antihistamine effects

oropharyngeal dryness - food bolus not formed > increased aspiration

body habitus is not strongly associated with an increased risk of CAP.

dementia (eg, Alzheimer), which increases the risk of aspiration due to impaired ability to
protect the airway and coordinate the muscles of swallowing

Can thrombectomy done if thrombolysis is not indicated in pt with


ischemic stroke ?
Thrombectomy may be used irrespective of wether they they qualify for thrombolysis

Because thrombolysis and mechanical thrombectomy are considered independently of each


other, a specific patient may be eligible for one, both, or neither.

For patients with ischemic stroke, eligibility for thrombolysis - <4.5 hr and mechanical


thrombectomy - <24 hr is considered independently.

SO CHECK FOR CONTRAINDICATIONS FOR BOTH INDEPENDENTLY - ADMIN BOTH /


JUST ONE /NEITHER

Pt with Diabetic neuropathy , urinary incontinence . Now has memory loss


and disorientation + dementia since one month . NBSIM ?
adverse effect of polypharmacy , possibly amitriptyline / oxybutinine , anticholinergic meds >
cognitive impairment

eligibility criteria for Mechanical thrombectomy ?


CT angiography - flow void in MCA / large vessel

noncontrast CT - hyperdense large vessel

CT angio head - imaging shows a large mismatch between core and


penumbra. significance ?
core (brain likely irreversibly infarcted) and the penumbra (hypoperfused brain at risk for
infarction)

indicates area of reversible ischemia and penumbra tissue may be salvaged

Does parkinsons disease always present with tremor ?


NO, idiopathic Parkinson disease can also manifest with primarily bulbar symptoms

Nervous system -DONE 58


Bulbar s/s in parkinsons disease ?
1.Hypophonia (a soft voice)

2.Aspiration of liquids (coughing when drinking)

3.Loss of olfactory function (often perceived by patients as change in taste)

4.Rigidity of the upper limbs( increased passive resistance )

5.Stooped posture and slow gait

Amyotropic lateral sclerosis VS parkinsons disease ?


both can present with predominantly bulbar symptoms such as dysphagia and dysarthria

posture - normal ( stooped )

Muscle s/s ? - both upper and lower motor neuron signs ( rigidity , tremor )

loss of olfactory fx ? absent ( present )

STIFF PERSON SYNDROME - autoimmune condition - presentation ?


rigidity of axial muscles leading to gait disturbance

severe, painful muscle spasms ppt by loud noise causing a fall

common in women , pt have comorbid type 1 diabetes

Most cases of nerve root compression are due to?


vertebral disc herniation

REMEMBER - patient-identified inciting event (eg, heavy lifting) is present in only a minority of
case

spondylolisthesis VS vertebral disc herniation


both have radicular pain and neurologic deficits

onset of back pain? occurs gradually, with weeks to months (vs a day)

thickened ligamentum flavum VS vertebral disc herniation?


onset of symptoms? bilateral and develop over months to years (vs unilateral , 1 day)

s/s Exacerbated by ? walking (vs coughing or sneezing)

The major cause of death in patients with advanced Parkinson disease?


bacterial aspiration pneumonia.

NBSIM ?

swallow evaluation is required as the disease progresses

alter food consistency or eating position to reduce the risk of aspiration pneumonia.

Nervous system -DONE 59


Surgery
Malignant hyperthermia vs NMS ?
both cause rigidity, myoglobinuria, tachypnea

Cause and onset ?

onset within minutes during or shortly after anaes, mortality high in 24 hrs (onset over
days s/s persists for days postoperatively)

Cause - succinylcholine or a volatile anesthetic eg, halothane(neuroleptic agent eg,


haloperidol, fluphenazine)

Why is MRI not done after seeing clinical s/s of lumbosacral radiculopathy
?
because nearly all cases are due to benign etiologies and symptoms are typically self-
limiting

When is MRI used for dx in case of ANY Radiculopathy ?


severe, progressive, or bilateral neurologic deficits (eg, saddle anesthesia, urinary retention,
severe weakness)

high concern for malignancy or epidural abscess (eg, fever, intravenous drug use).

Pt with Uncomplicated lumbosacral radiculopathy ie just radiating pain ,


no neurological deficits . Next step ?
Nonsteroidal anti-inflammatory drugs and acetaminophen are the preferred first-line
drugs.

In uncomplicated LSR > initial management focus> alleviating symptoms.

Amyotrophic lateral sclerosis vs Spinal cord compression ( eg Cervical


spinal cord )
Both causes - UMN and LMN signs

asymmetric limb weakness and atrophy (symmetric)

Lhermitte sign - atypical ( common in cervical spondylotic myelopathy)

neck pain uncommon as cause is neurodegen. ( common in cervical spondylosis causing


spinal cord compression > pain)

Lhermitte sign (electric shock–like pain with neck flexion)

Multiple sclerosis vs Spinal cord compression/ Cervical spondylosis?


Both cause Lhermitte sign and UMN lesions

Nervous system -DONE 60


AFFECTS CNS , doesnt affect LMN as doesnt affect PNS ( LMN affected )

How does cervical radiculopathy present ?


pain in the neck radiating to the shoulder/arm with neck movement

weakness in an upper extremity myotome

sensory loss in an upper extremity dermatome

Dx of cervical radiculopathy ?

usually made clinically , but MRI - cervical spine if severe progressive and b/l deficits

2 common outcomes of cervical spondylosis ?


Cervical spondylosis is marked by cervical spine degeneration

1.Cervical radiculopathy

Mechanism = degeneration and osteophyte formation - intervertebral foramen narrowing


> compressive nerve root symptoms

s/s >weakness and sensory loss in respective dermatome

2. compressive cervical myelopathy

Mechanism : - Degeneration and thickening of the lateral vertebral bodies and posterior
longitudinal ligament > spinal canal narrowing and subsequent spinal cord compression

S/s - neck pain - LMNs in UE and UMN in LE , bowel/bladder dysfunction

Cervical myelopathy is known as radiculomyelopathy because


1.myelopathic symptoms (eg, upper motor neuron signs below the lesion) and

2.radicular symptoms (eg, lower motor neuron signs, pain in a dermatomal/myotomal pattern).

Does normal pressure hydrocephalus always occur in elderly ?


NO , may occur in any age group due to neurologic trauma, infection, or subarachnoid
hemorrhage resulting in scarring of the arachnoid granulations

IS dementia in pts with NPH reversible ?

Yes , reversible

Pt with intracerebral hemorrhage. Rt side weakness . Now after sometime


> minimally responsive +hyperextension of all four extremities.Dx ?
continued hemorrhage expansion causing brain herniation (eg, midline shift on CT scan,
decerebrate posturing)

Mx ?

1.ABCs (Airway, Breathing, Circulation) should be reassessed.

2.immediate intubation and mechanical ventilation (airway and breathing of the ABCs)

Nervous system -DONE 61


followed by mx to prevent bleeding and lower ICP

Why does brain herniation require immediate intubation and mechanical


ventilation ?
AS brain herniation can lead to respiratory failure due to

1.inability to protect the airway.- due to decreased consciousness

2. airway protective reflexes eg, cough, gag - impaired due to dysfuntion of cranial nerve
nuclei

3.decreased ventilatory drive - due to brainstem compression > hypercarbia and


hypoxia.

ABCs (Airway, Breathing, Circulation) should be reassessed. in any patient with rapid change


in clinical status

The most common cause of cervical myelopathy in older adults?


spondylosis

Examination shows - LMN sigs in arms and UMN signs in legs

Syringomyelia vs Cervical spondylosis in 57 year man


Neuro findings = Loss of pain and temp in cape distri ( LMN signs in arms , UMN in legs )

epidemio = less common ( very common )

Lhermitte sign occurs when?


neck flexion compresses and activates the ascending spinothalamic pain tracts

Nonspecific sign seen in multiple CNS diseases

1. multiple sclerosis

2. transverse myelitis

3. Cervical spondylosis

Does Arteriovenous malformations always occur in children ?


NO, can occur in anyone before age 40 , congenital or de novo formation occurs

AVMs occur when an artery directly anastomoses with the veins without an interposed
capillary bed.

Arteriovenous malformation vs Hypertensive vasculopathy


both cause Intracerebral hemorrhage

1.recurrent U/L headache ( not recurrent and not U/L )

2.h/o of HTN ( h/o of prolonged HTN )

Nervous system -DONE 62


Arteriovenous malformation vs Cerebral amyloid angiopathy
Both cause Intracerebral hemorrhage

Young ie <40 with recurrent headache and seizures (elderly patients and is associated with
dementia)

How does Intracerebral hemorrhage (ICH) present ?


progressive headache, nausea/vomiting, and altered mental status over a period of
minutes to hours.

How does Arteriovenous malformations present ?


recurrent headache, seizure, or focal neurologic deficit (due to compression).

can cause spontaneous bleed

Which s/s point towards a cerebellopontine tumour as cause of Bells palsy


?
Slowly progressive facial weakness associated with hearing loss and facial twitching

concerning for neoplastic growth with ongoing irritation of the facial nerve (CN VII)

Most commonly affected vertebral bodies in cervical facet dislocation ?


C5/C6, which lead to C6 radiculopathy

C6/C7, which lead to C7 radiculopathy.

CT scan of Cervical facet dislocation vs Cervical stenosis?


anterior subluxation of the vertebral body relative to lower vertebral body ( narrowing of the
spinal canal.)

Cervical facet dislocation vs Burst fracture ?


Both can occur after trauma

Location - typically at C5/C6 or C6/C7 ( typically occurs at L1)

CT - anterior subluxation of the C6 vertebral body relative to C7 ( burst vertebral body)

Does cervical strain cause focal neurologic deficits?


No , neither does it cause anterior dislocation of vertebral bodies

what type of injury causes Cervical facet dislocation ?


forced flexion of the cervical spine (eg, falling onto a flexed neck) > single facet is usually
dislocated and results in radiculopathy of the corresponding nerve root.

Mx of cervical radiculopathy ?
Since Most patients experience gradual resolution

Nervous system -DONE 63


Symptomatic Mx - NSAIDS and avoidance of provocative maneuvers.

glucocorts added for severe pain

Maintain moderate physical activity , STRICT BEDREST - AVOIDED

Cervical radiculopathy pt - fails to improve on NSAIDs and oral glucocorts


. NBSIM ?
Epidural corticosteroid injections

Is it necessary for triad to be present for dx of normal pressure


hydrocephalus ?
1.all 3 features are not required for diagnosis

2. only gait dysfunction, typically the most prominent symptom, is mandatory. for DX

triad of urinary incontinence, cognitive impairment, and gait abnormalities

Role of glucocorts in hemorrhagic or ischemic stroke ?


No role - used to reduce inflammation and peritumor edema in intracranial tumours

Tumor vs hemorrhagic stroke - CT ?


less hyperdense and demonstrate peritumor edema and central necrosis ( hyperdense )

Does cerebellar hemorrhage present with hemiparesis and sensory loss ?


Very rare , contrast to other strokes where it is very common

Spinal epidural abscess > progressive neurological symptoms in following


sequence ?
focal back pain → nerve root pain(eg shooting or electric shock ) → motor weakness, sensory
changes, and bowel/bladder dysregulation → paralysis

Does spinal epidural abscess always present with triad fever, spinal pain,


and neurologic symptoms?
No , not always , suspicion should be high , MRI as early dx prevents paralysis and death

Management of hemorrhagic stroke after assessment of the "ABCs" (ie,


airway, breathing, circulation)
1. Blood pressure control

Reversible and titratable antihypertensive medications such as intravenous


nicardipine or labetalol are used

SBP goal - 140-160 mm Hg

2. Reversal of anticoagulation -Targeted reversal of any anticoagulation

Nervous system -DONE 64


vitamin K is typically given to patients on warfarin

protamine sulfate to patients on heparin.

3. Regulation of ICP

Increased ICP is treated with interventions such as head-of-the-bed elevation, sedation,


and osmotic therapy (eg, mannitol).

When is surgical evacuation of hematoma due to hemorrhagic stroke done


?
1.evidence of impending brain herniation (eg, large hematoma with midline shift on CT scan)

2.or elevated ICP that is refractory to medical management

Most commonly injured nerve in anterior shoulder dislocation ?


axillary nerve - innervates the teres minor and deltoid (weakened shoulder abduction)

sensory innervation to the skin overlying the lateral shoulder

How does Meningiomas appear on MRI ? Benign or malignant


BENIGN , extra-axial well-circumscribed or round homogenously enhancing dural-based
mass that is partially calcified

Presentation of meningiomas ? Management ?

headache, seizure, and focal neurologic deficits due to mass effect.

complete Surgical resection

more commonly found in middle-age to elderly women

Anterior cerebral artery stroke vs Rheumatoid arthritis cervical


myelopathy ?
both cause extremity weakness with spastic paraparesis.

1.weakness onset ? U/L or B/L

maximum weakness upon presentation and the findings are unilateral (slow progressive
weakness , B/L findings)

2.dysarthria and aphasia common ( not seen)

Critical illness polyneuropathy vs Rheumatoid arthritis cervical


myelopathy
peripheral nerve injury results in hyporeflexia ( UMN - hyperreflexia seen )

Critical illness polyneuropathy is a complication of sepsis characterized by axonal injury of the


peripheral nerves.

Uremic polyneuropathy vs Carpal tunnel syndrome

Nervous system -DONE 65


both present in ESRD

s/s ? progressive pain and paresthesia in the feet ( CTS - hands )

Effect of dialysis resolves when initiated ( worsens as more deposition of beta-2 microglobulin)

Pt with sudden-onset, severe headache and neck stiffness and fever


.NBSIM ?
non contrast CT scan

1.evaluate for SAH first because it has high mortality

2. CT less invasive than LP

3. IF CT negative evaluate via LP - meningitis / SAH

Meningitis and Subarachnoid hemorrhage both presents with fever, neck


stiffness, and severe headache
fever - High grade fever with chills ( low grade fever , no chills )

Onset - relatively slow onset (rapid-onset headache and progressive s/s )

Which pts are susceptible to copper deficiency ? x3


1.excessive zinc ingestion-  zinc competes with copper for absorption

2.history of gastric surgery (eg, bariatric)

3.chronic malabsorption (eg, inflammatory bowel disease)

Hematologic abnormality in copper deficiency ?


1.Hypochromic microcytic anemia - due to impairment of iron absorption - macrocytic /
normocytic may present

2. Leukopenia may occur

Mx of Copper deficiency ?

copper supplementation and discontinuation of zinc.

Dx of copper deficiency ?

low serum copper and ceruloplasmin levels

B/L sensorineural hearing loss associated with imbalance . family h/o of


hearing loss. - Dx ?
bilateral vestibular schwannomas - characteristic of neurofibromatosis type II (NF2).

Sporadic vestibular schwannomas vs B/L vestibular schwannomas


Sporadic vestibular schwannomas vs B/L vestibular schwannomas

sporadic ( assosciated with Neurofibromatosis type 2 )

Nervous system -DONE 66


Vestibular schwannoma primarily affects the
1. cochlear( sensorineural hearing loss)  and
2. vestibular nerves.( imbalance, particularly when patients are deprived of visual input)

What happens in neurogenic shock ?


Lesions that arise above T1

interruption of the descending sympathetic


fibers - unopposed parasympathetic stimulation

leads to vessels (hypotension) and heart (bradycardia) and hypothermia due to lack of


peripheral vasoconstriction

preganglionic sympathetic neurons in the lateral horn of the spinal cord (at levels T1-L2).

Acute spinal cord injury is generally associated with an initial period


(several minutes) of?
massive sympathetic stimulation (leading to hypertension and tachycardia) due to release of
NE from the adrenal glands.

Acute lumbosacral radiculopathy vs Cauda equina syndrome


both caused by disc herniation and cause low back pain that radiates.

saddle anesthesia and bowel/bladder dysfunction are generally absent ( present - S3-S5 roots
-rectal sphincter paralysis, Saddle anesthesia impingement of S2-S4 nerve roots)

Spinal cord injury above T6 can be complicated by?


autonomic dysreflexia, in which noxious stimuli below the injury level trigger an unregulated
sympathetic response, leading to severe hypertension.

Why is there bradycardia in autonomic dysreflexia ?

A compensatory parasympathetic response above the lesion typically


causes bradycardia

no compensatory parasymp below the level

Mx of autonomic dysreflexia ?

1.Upright position for orthostatic blood pressure reduction

2.removing noxious stimuli eg urine retention

3.treating the hypertension. eg nitrates

Which cranial nerve is often involved in rapid hematoma expansion after


head injury ?
1.increased ICP > uncal herniation

2.ipsilateral fixed and dilated pupil due to oculomotor nerve (CN III) compression

Nervous system -DONE 67


How does uncal herniation present ?
1. Ipsilateral fixed and dilated pupil -Paralysis of the oculomotor muscles occurs later -
ptosis and down and out position of eyes

2.Contralateral hemiparesis - due to compression of ipsilateral cerebral peduncle of mid


brain

3.Contralateral homonymous hemianopsia with macular sparing - due to compression of


ipsilateral PCA

Spinal epidural hematoma is a complication of which procedures ?


1.neuraxial anesthesia (eg, epidural block)

2.lumbar puncture

3.spinal surgery

ESP IN OLDER ADULTS TAKING ANTITHROMBOTIC MEDS ( even if discont.


periprocedurally)

Mx of spinal epidural hematoma ?

urgent MRI and neurosurgical decompression eg laminectomy

Additional risk factors include other coagulopathies (eg, thrombocytopenia), spinal


structural abnormalities, and osteoporosis.

How does spinal epidural hematoma present


1. slowly progressive ie days - as bleeding is from venous source

2. Localized back pain and point tenderness

3.progressive motor and sensory dysfunction in affected nerve root

4.bowel and bladder dysfunction may occur - Cauda equina syndrome

Apart from ingestion of spores/ foodborne botulism , how can you get
botulism ?
Wound botulism - Clostridium botulinum spores contaminate a puncture injury (eg,
intravenous needle), germinate, and generate neurotoxin in vivo.

How does wound botulism differ from infant botulism ?


fever and leukocytosis 10 days after transmission ( absent )

How does wound botulism present ?


1.Symmetric, descending motor paresis beginning with the CNs

CN III, CN IV, and CN VI - diplopia, ptosis, mydriasis

CN IX and CN X - dysphagia, suppressed gag

Nervous system -DONE 68


2.Progressive respiratory compromise due to diaphragmatic paralysis

3.Autonomic dysfunction - eg, ileus, orthostatic hypotension, urinary retention

When botulism is suspected . NBSIM ?


Urgent treatment with equine botulinum antitoxin is required and should not be delayed for
diagnostic evaluation.

Confirmation requires the isolation of C botulinum in culture or identification of toxin in serum

Complication of thoracic aortic aneurysm repair on spinal cord ?


spinal cord ischemia, especially of the anterior cord.

Also causes urinary retention - from damage to the descending autonomic tracts involved in
bladder control

Anterior cord syndrome vs central cord syndrome ?


both have similar loss of pain/temperature sensation

bilateral weakness that is proportionate in the UE and LE ( disproportionate upper extremity


weakness)

Cause of central cord syndrome ?


1. Hyperextension injury, especially in elderly patients with cervical spine degenerative
changes eg cervical sponylosis

2. syringomyelia - slowly progressive

sacral (eg, bowel/bladder) and lower extremity function is generally preserved.


a hyperextension injury to the neck (eg, whiplash due to rear-end collision)

Does glucocorticoids help reduce vasogenic edema in traumatic brain


injury and decrease ICP ?
no , it just increases the risk of death in pts with traumatic brain injury

initial treatment of elevated intracranial pressure in patients with traumatic


brain injury?
Osmotic therapy (eg, hypertonic saline, mannitol)

reducing parenchymal volume and overall intracranial pressure.

For patients with elevated ICP refractory to initial reduction measures?


NBSIM ?
decompressive craniectomy

craniectomy removes a substantial portion of the skull, allowing expansion of the brain, which
rapidly reduces ICP.

Nervous system -DONE 69


Can elevated ICP be treated with lumbar puncture ?
CSF drainage is typically performed via an - external ventricular drain . LP - avoided
- because it may cause an acute pressure gradient - risk for cerebral herniation

acute pressure gradient (ie, high intracranial to low lumbar)

How does traumatic brain injury / any trauma cause coagulopathy ?


increased activation of the coagulation pathway (due to severe tissue injury) > uncontrolled
coagulation and consumption of coagulation factors

1. Hypocoagulability from consumptive coagulopathy

2. Hyperfibrinolysis - breaks down clots

ALS vs foramen magnum meningioma


Neuro findings ? asymmetric muscle weakness with both UMN and LMN signs ( symmetric
weakness and absent LMN , only UMN )

Central cord syndrome manifestations ?


primarily develop upper extremity manifestations, including

1.Weakness - alpha motor neuron damage

2. Pain, temperature, and sensory loss in dermatomes at and surrounding the level of


injury - damage to the posterior grey column.

3.Reflex loss (eg, tricep reflex) - at the level - fibers that cross from dorsal to ventral horn
damaged

Due to the more central location of the lesion, the lateral spinal tracts running to the sacrum
(eg, bowel, bladder) and lower limbs are generally spared.

In patients with traumatic spinal cord injury. Why is bladder catheterization


required ?
because disruption of the autonomic tracts involved in bladder control can lead to urinary
retention.

performed to prevent bladder distension and possible injury.

Meningioma - tumour origin ? = arachnoid cap cells

Syringomyelia is most commonly asso with ?


1.Arnold-Chiari type 1 malformations (extension of the cerebellar tonsils into the foramen
magnum)

2.sequela of meningitis

3.inflammatory disorders

4.tumors, and trauma.

Nervous system -DONE 70


When does syringomyelia develop after initial insult ?
Symptoms may present months to years after the initial insult.

Syringomyelia can occur at any level but most commonly involves the cervical or thoracic
spine, typically resulting in upper extremity symptoms

syringomyelia, a disorder caused by disruption of CSF drainage from the central canal >
leading to formation of a fluid-filled cavity (syrinx)

Dx of syringomyelia ? Mx of syringomyelia ?
MRI - shows intramedullary cavity = surgical intervention (eg, shunt placement).

Does amyotrophic lateral sclerosis have loss of sensory function ?


no loss of sensory function , only UMN/LMN symptoms

What is emergence delirium ?


A hyperactive state (eg, agitation, disinhibition) that usually manifests in the operating room
but sometimes develops after initial normal emergence (eg, within 45 minutes).

Which age group is susceptible to emergence delirium ?

common in children

Which surgeries and what h/o in adults predisposed to emergence delirium ?

after abdominal/breast surgery and in patients with a psychiatric history (eg,


posttraumatic stress disorder).

All patients who sustain orbital trauma should undergo assessment of?
visual acuity and extraocular movements.

CT scan is obtained if there is evidence of a fracture , changes in vision or extraocular


movements, or the examination is limited due to soft-tissue swelling or altered mental status.

Subfalcine herniation, a type of brain herniation


occurs when the cingulate gyrus is displaced under the falx cerebri.

does not cause pupillary involvement but may cause ipsilateral anterior cerebral artery
compression > contralateral leg weakness.

s/s indicative of brain tumor ?


1.Headaches(classically worse with recumbency)

2.increased intracranial pressure

3.unprovoked first seizure

How does frontal lobe tumours present ? x3

Nervous system -DONE 71


personality changes

abulia

anhedonia.

Frontotemporal dementia vs Frontal lobe tumors?


both cause personality changes
headaches and papilledema - absent ( present )

Does frontal lobe tumour always cause disinhibition and impulsiveness ?


can also lead to

1.decreased motivation and/or

2.lack of concern regarding the diagnosis (ie, abulia)

3.loss of interest (ie, anhedonia)

Outcome of Mesial temporal (ie, hippocampal) lobe sclerosis?


most common cause of focal epilepsy, which can subsequently generalize

preceded by a distinct aura eg epigastric tenderness, olfactory hallucinations

No change in intracranial pressure , no personality changes

Features that suggest cocaine as cause of intracerebral hemorrhage ?


1.Young age: Most patients who sustain stroke are age >60.

2.Absence of typical risk factors: eg Chronic hypertension

3.Sympathetic activation - tachy , mydriasis and hyperthermia

4.Location: - subcortical locations eg, thalamus & intraventri. hemorrhage

most common underlying cause of ICH is chronic hypertension

hyperthermia in cocaine use due to cocaine-induced vasoconstriction preventing heat


dissipation

When is thrombin time measures in case of intracerebral hemorrhage ?


only recommended in patients with ICH who are taking direct oral anticoagulants (eg
rivaroxaban)

Any 1 of the following is an indication for cervical spine imaging


1.Neurologic deficit

2.Spinal tenderness

3.Altered mental status eg somnolence

4.Intoxication

Nervous system -DONE 72


5.Distracting injury - ( eg open right tibia-fibula fracture)

6.high-energy mechanism of injury eg high-speed motor vehicle collision, fall ≥3 m [10 ft],
trauma causing concomitant closed-head injury

CT scan without contrast of the cervical spine is the preferred screening test to evaluate for


cervical spine injury - Ct more sensitive than radiographs

high-speed motor vehicle collision . Gross deformity in leg. capillary refill


<2 second . NBSIM ?
CT scan of the cervical spine , CT angio LE not done as capillary refill normal

Complete pulse examination and measurement of the injured extremity index BETTER THAN
CT ANGIO.

presence of a single vertebral fracture in a patient with blunt trauma .


NBSIM ?
>indication to image the entire spine.via CT scan

because the risk of a second, noncontiguous vertebral fracture is as high as 20%.

paroxysmal sympathetic hyperactivity (PSH) seen in which pts and


mechanism ?
1.in pts with severe traumatic brain injury

2.occurs following damage to cortical areas that are responsible for modulating


and inhibiting lower sympathetic centers - eg, in the hypothalamus, brainstem, and spinal
cord)

How does paroxysmal sympathetic hyperactivity (PSH) present ?


rapid-onset episodes of tachycardia, hypertension, and tachypnea often accompanied by
fever and diaphoresis

lasts 20-30 minutes

triggered by ? = external stimuli (eg, bathing, repositioning) / spontaneously.

Mx of paroxysmal sympathetic hyperactivity (PSH)?


supportive care (eg, avoiding triggers, treating fever)

medication (eg, opioids, GABA-agonists, α2agonists) to abort and/or prevent episodes

PSH is a clinical diagnosis with no confirmatory test , Opioids may improve symptoms via a
general reduction of sympathetic tone.

Uncontrolled acute pain vs Paroxysmal sympathetic hyperactivity?


Both cause episodic tachycardia, hypertension, and tachypnea

both improve with opioids

Nervous system -DONE 73


vital sign changes - less extreme ( very extreme )

fever - uncommon ( common with diaphoresis )

Eg parasympathetic hyperactivity pts > Temperature is 38.8 C (101.8 F), blood pressure is
194/110 mm Hg, pulse is 146/min, and respirations are 40/min.

PEDIATRICS
can infantile botulism occur only due to honey ingestion ?
ingestion of honey or dust contaminated with Clostridium botulinum spores.

Classically presents with hypotonia and descending paralysis.

Cephalohematoma (subperiosteal scalp swelling) vs intracranial


hemorrhage
Prognosis resolves spontaneously without sequelae ( UMN injury )

Epidural hematoma vs Subdural hematoma


Cause - tear of middle meningeal artery rapid expansion ( tearing of bridging veins - gradual
bleed )

Epidemio - trauma at pterion , young adults ( older adults esp those on antithrombotic


agents/chronic alcoholics )

Coma - Brief loss of consciousness followed by lucid interval ( coma at time of onset)

Similarly Diffuse axonal injury also presents with immediate coma

Optic pathway gliomas is asso with genetic condition ?


Neurofibromatosis type 1 - café-au-lait macules, Axillary freckling , Lisch nodules, and
neurofibromas

How does optic pathway gliomas present ?

unilateral vision loss, proptosis, esotropia, and optic disc pallor

headache and decreased visual acuity

If it grows large enough - decreased visual acuity, alterations in color vision, optic nerve
atrophy, and proptosis.

Most common aura found in migraine in children ?


Visual auras - eg scintillating scotoma or bright flashing lights - MC

Sensory aura eg unilateral numbness or weaknes

Auras last ? <1 hour, and patients are neurologically intact afterward.

Nervous system -DONE 74


Refractory migraine headaches( ie unresponsive to supportive mx &
analgesics ) MX ?
1.triptans oral or intranasal - serotonin receptor (5-HT1B/1D) agonist that decreases
neurogenic inflammation and CGRP release

2. OR Antiemetics (eg, promethazine) , dihydroergotamine

Triggers of migraine ?
stress, fasting, dehydration, menses, and sleep deprivation

Pathogenesis involves abnormal trigeminovascular system activation and release of calcitonin


gene-related peptide (CGRP), a neuropeptide involved in pain signaling.

Pathogenesis of concussion ?
Head trauma > widespread neuron depolarization > decreased cerebral blood flow, and
localized lactic acidosis > leads to transient disturbance of normal neuronal function

OR axonal shearing from rotational acceleration of the brain after a fall or strike to the
head.

Typical symptoms of concussion ?


1.headache, dizziness, disorientation, and/or amnesia, which may be retrograde or
anterograde.

2.transient abnormalities in coordination (eg, stumbling, falls), speech (eg, slurring), attention
(eg, poor focus), or emotion (eg, lability).

vomiting and loss of consciousness may also occur.

Diffuse axonal injury vs Concussion


Both are due to traumatic brain injury

Mechanism - tearing of the white matter tracts ( transient disturbance of normal neuronal
function.)

S/s - severely obtunded or comatose.( headache, dizziness, disorientation, and/or amnesia)

Does idiopathic intracranial hypertension (IIH) , occur in children ?


Remember that it can also occur in nonobese children

IIH most commonly occurs in obese women of childbearing age

How does idiopathic intracranial hypertension present in prepubertal


population ?
headache may be less obvious

vision abnormalities (eg, blurry vision, enlarged blind spot) are often predominant

Nervous system -DONE 75


Increased ICP > on the optic disc causes swelling (papilledema) > blurry vision ( doesnt
improve with refraction )&  enlarged blind spot.

Which cranial nerve is more commonly affected in pt with increased ICP?


abducens nerve (CN VI) because of its long course through the skull to the lateral rectus
muscle. > diplopia and impaired eye abduction

First step in evaluation for increased ICP , papilledema in children ?


neuroimaging to rule out anatomic causes of increased ICP

if negative - LP done, elevated opening pressure is dx of idio intra HTN

Mx of idio intracranial HTN with acetazolamide helps prevent?


prevent permanent vision loss.

Glycogen storage diseases vs Galactosemia


both present with hepatomegaly and signs of hypoglycemia (eg, seizures)

ONSET ? occurs during or after age 3-6 months ( presents in first few days of life. )

GSD - hypoglycemic eps do not typically occur until age 3-6 months because they are caused
by periods of fasting > begin when the infant sleeps longer through the night.

Initial evaluation in children with hydrocephalus ?


ultrasound of the head if with an open fontanelle

MRI of the brain if with a closed fontanelle.

Rett syndrome vs Hydrocephalus


Head growth ? slowed head growth and microcephaly are typical.( rapidly enlarging head
circumference eg crosses two major percentiles 25% 50% in months )

Spinal muscular atrophy (Werdnig-Hoffmann disease) vs Botulism ?


pupils - does not affect the pupils ( sluggish pupil )

weakness - greater in LE than UE ( generalised )

Autonomic s/s - absent ( constipation , drooling etc present )

Foodborne botulism ( preformed toxin ) vs infant botulism (ingestion of


spores)
prodrome ? nausea, vomiting, abdominal pain, and diarrhea. ( absent )

ie prodrome before descending flaccid paralysis

Pt with suspected Neurofibromatosis type 1 . NBSIM ?

Nervous system -DONE 76


NF1- at risk for developing intracranial neoplasms - MC Optic pathway glioma > MRI of the
brain and orbits to evaluate the intracranial structures.

Headache is often worse in the morning because intracranial pressure increases


overnight while supine

increased risk for astrocytomas and brainstem gliomas, even into adulthood.

Acute cerebellar ataxia in pt with no h/o of neuro disorder ? Px ?


following an infection (eg, varicella infection), resolve within 2 weeks.

Presentation of acute cerebellar ataxia ?

acute onset (within hours or days) of ataxia, nystagmus, and dysarthria

vibration and propioception , reflexes intact

Advice given to pt with concussion ?


1.Symptom management (eg, NSAIDs, ondansetron)

2.physical and cognitive rest for 24-48 hours

3.gradual return-to-play over 5 days with increasing intensity

pt should remain symptom free before progressing to next level

increasing intensity of physical activity (eg, light exercise → noncontact sport →


competitive play).

Most patients resume normal activities within a month.

measles, mumps, and rubella (MMR) vaccine


recommended at age 1 and again at age 4.

increased incidence of sudden death in Rett Syndrome .Mechanism ?


cardiac electrical instability from autonomic dysregulation

Angelman syndrome vs Rett syndrome ?


defect? - absence of the maternally inherited copy of the UBE3A gene ( MECP2 gene
mutations)

Development ? developmental delay ( dev. regression)

resp abnorm? - absent ( hyperventilation-hypoventilation episodes)

Mx of homocystinuria ?
vitamin B6, folate, and vitamin B12 to lower homocysteine levels

antiplatelets or anticoagulation should be administered to prevent strok , coronary art. disease


, venous thromboemb.

Nervous system -DONE 77


Ehlers danlos Vs marfans/ homocyst
both have joint laxity, and skin hyperelasticity.

tallstature ? absent ( present )

lens dislocation ? absent ( present )

hypercoag? absent ( present in homocsyt)

How does shaken baby syndrome present ?


1.Repetitive acceleration-deceleration> shearing of the bridging veins > subdural bleeding

2.vitreoretinal traction & retinal hemorrhages

Shaken baby syndrome vs Severe intraventricular hemorrhage


both can have signs of hydrocephalus (eg, seizures, bulging fontanelle)

onset ? after trauma ( presents in premature infants within the first few days of life.)

Retinitis pigmentosa vs Vitamin A deficiency ?


Both cause night blindness

cornea/conjunctiva ? normal( excessive dryness of the cornea and conjunctivae)

If patient eats a variety of vegetables, diet contains vitamin A.

Chiari malformations
Chiari 1 malformation - When only the cerebellar tonsils are displaced

Chiari II malformation - herniation of the cerebellar tonsils and vermis as well as inferior
displacement of medulla.

Chiari III malformation - severe and involve herniation into an encephalocele.

Only sign of syringomyelia in children? NBSIM ?


Scoliosis only - if present > MRI to dx chiari 1 malformation

How does chiari 1 malformation present ?


asymptomatic throughout childhood.

present in adolescence or early adulthood with occipital headache and/or neck pain

pain ppt by ? - physical activity or Valsalva maneuvers (eg, cough, straining) due to


pressure of the cerebellar tonsils on the foramen magnum

Most common asso with chiari 1 malformation ?


Syringomyelia - mc in cervical region & presents with progressive neurologic dysfunction (eg,
paresthesia, weakness)

Nervous system -DONE 78


Guillain-Barré syndrome affects which part of the nervous system ?
immune-mediated polyneuropathy caused by demyelination of peripheral nerve fibers.

Motor nerves are most commonly affected, but sensory and autonomic nerves may also be
involved.

Does GBS pt have autonomic s/s ?


YES , autonomic symptoms (eg, blood pressure/heart rate instability, ileus) often parallel the
severity of motor deficits.

Cyanotic ep of Tetralogy of fallot vs breath holding spell ?


Both can present after crying

breathing ? - rapid and deep breathing, & systolic ejection murmur due to RVOT ( breath-
holding only )

what kind of smoke exposure causes sudden infant death syndrome ?


maternal smoking during pregnancy

passive postnatal exposure

Impaired cardiovascular reflexes (eg, increased heart rate due to hypercarbia) and diminished
arousal responses may account for this elevated risk.

Is prone position advised for infants ?


Prone-sleep positioning - increased risk of SIDS

advised only during awake , observed periods

helps in gross motor development and positional plagiocephaly prevention

Is side sleep position advised for infants ?


avoided due to an increased ability to roll into the prone position

Does GE reflux in infants , warrant infant to be placed in prone position ?


aspiration risk is not greater in the supine position due to an intact gag reflex. > so supine is
advised

Sudden infant death syndrome


refers to the unexplained death of infants age <1.

unexpected death that cannot be explained by history or postmortem examination

Pacifier use during sleep is associated with a decreased risk of Sudden Infant Death
Syndrome.

Nervous system -DONE 79


neonates born at <32 weeks gestation are at highest risk for intravent.
hemorrhage because?
germinal matrix involutes by week 32.

When does Intraventricular hemorrhage occur ? Mx ?

occurs in the first 3-4 days of life

mostly symptomatic (eg, BP stabilization, antiseizure medication)

all preterm neonates born at <32 weeks gestation require screening head
ultrasound. WHY ?
Because Intra Ventricular Hemorrhage can be asymptomatic , can cause complications if
not treated eg posthemorrhagic ventricular dilation

Risks in preterm infants and screening req? x5


necrotizing enterocolitis (NEC) - screening not reqd

patent ductus arteriosus - not reqd

karyotyping - not reqd unless cf present

sepsis - not reqd unless cf present

Intravent hemorrhage - A/S so screening head USG

How does infantile spasms present ? ASSO?


clusters of brief muscle contractions - appears similar to exaggerated startles (eg, flexion of
neck) and abdominal crunches (eg, flexion of trunk and extremities)

Mc asso with tuberous sclerosis but can occur in isolation

intracranial Tumours and their assosciations ?


Subependymal giant cell tumor - Tuberous sclerosis complex (TSC)

Acoustic schwannoma - NF2

cerebellar hemangioblastoma - Von Hippel-Lindau disease

Leptomeningeal angiomatosis - Sturge-Weber syndrome.

Optic nerve glioma - NF1

what ppts intraventricular hemorrhage in premature infants ?


acute changes in cerebral perfusion (eg, hypotension, hypo- or hyperventilation, increased
venous pressure)

Long-term sequelae of intraventricular hemorrhage ?

cerebral palsy and intellectual disability

Signs of benign macrocephaly ?

Nervous system -DONE 80


1.Normal development

2. NO syndromic features

3. NOsigns of increased intracranial pressure (eg, bulging fontanelle)

4.NO signs of infection (eg, fever, lethargy)

Mx of positional plagiocephaly, which is seen as occipital flattening?


cranial molding helmet

Most common cause of macrocephaly ? NBSIM ?


benign familial macrocephaly due to megaloencephaly (increased brain parenchyma volume)

Head ultrasound - if abnorm - MRI head

benign familial macrocephaly - h/o of large head in first degree relative

coexisting otitis media and mastoiditis, now has nocturnal headaches and
morning vomiting . Explain ?
temporal brain abscess from primary infection

ICP increases in supine due to gravity - nocturnal headaches

Increased ICP stimulates medullary vomiting center and the area postrema

Can skull x ray be used for dx of intracranial pathology ?


Skull x-ray is useful for detecting fractures

not useful for evaluating brain tissue

most common type of central nervous system tumor in children ?


low-grade astrocytomas

initial presentation of a tumor originating in the cerebral cortex.?

New-onset seizures

High grade astrocytoma vs low-grade astrocytoma in children?


growth ? malignant fast growing ( slow-growing and benign )

course - symptoms over days to weeks ( s/s over months )

epidemio - rare ( most common )

Pediatric intracranial tumours and location ?


Craniopharyngiomas - sella turcica

Ependymomas - fourth ventricle in the posterior fossa

Medulloblastoma - cerebellar vermis- causes ataxia

Nervous system -DONE 81


Neuroblastomas - abdominal mass - arise from sympathetic ganglion cells - can mets to brain
- METS LESS COMMON

REMEMBER : - primary pediatric CNS tumors are much more common than CNS
metastases

minor head trauma and no high-risk features for intracranial injury in child.
NBSIM ?
Reassurance and discharge home with education regarding symptoms (eg, severe
headache) that would require reevaluation.

PT with sudden LOC . woke within minutes. Now drowsy and confused .
Does it indicate syncope since woke in minutes ?
NO , because syncope has spontaneous return to baseline neurologic functioning, no
subsequent confusion, ITS SEIZURE

seizures have delayed return to baseline - postictal state of transient confusion , lethargy,


and/or focal neurologic deficits

new-onset urinary incontinence + lumbosacral cutaneous findings (eg,


hair tuft). NBSIM ?
Closed spinal dysraphism (spina bifida occulta) SUSPECTED - Diagnosis is with MRI of the
spine

What is tethered cord syndrome ?


In some pts with spinal dysraphism , there is tight filum terminale > leading to stretching
of the distal spinal cord > LMN signs and symptoms

presence of UMN signs > negates the dx of tethered cord

What is tethered cord syndrome ?


In some pts with spinal dysraphism , there is tight filum terminale > leading to stretching
of the distal spinal cord > LMN signs and symptoms

presence of UMN signs > negates the dx of tethered cord

Cardiac rhabdomyoma progno in Tuberous sclerosis pts ?


large in the newborn period, the mass typically regresses spontaneously after infancy.

MC s/s of tuberous sclerosis


1.refractory epilepsy ( MC )

2.Intellectual disability and developmental delay esp prominent in those with ref. epilepsy

Cerebral palsy vs Lesch-Nyhan syndrome


Both can present with dystonia and spasticity

Nervous system -DONE 82


distinguishing feature ? self-mutilation in early childhood only seen in Lesch-Nyhan syndrome

Cerebral palsy is a nonprogressive motor disorder

all patients with bacterial meningitis esp in children . should undergo what


testing before hospital discharge ?
audiologic testing as soon as possible (ideally, before hospital discharge).

because Sensorineural hearing loss, most commonly seen post Streptococcus


pneumoniae meningitis, > due to inflammation of the cochlea and/or labyrinth

why is audiologic testing reqd ASAP in pts with bacterial meningitis?


Streptococcus pneumoniae meningitis > inflammation of the cochlea and/or labyrinth >
initially A/S > can rapidly progress to fibrosis and ossification of cochlea > irreversible
sensorineural hearing loss

once cochlear ossification present > cochlear implantation surgery becomes more difficult- SO
EARLY DX NECESSARY

Findings that are suggestive of benign microcephaly ? x3


1. Normal development ie no growth restriction

2. Absence of syndromic features eg downs

3.Mildly restricted head circumference

How does acute ischemic stroke in Young children (eg, age <6) differ from
older children / adults ?
nonlocalizing symptoms such as headache

generalized or focal seizures

altered mental status (eg, lethargy)

+ classic neurological deficits

Young children (eg, age <6) with suspected Acute Ischemic Stroke. choice
of dx ?
MRI with MR angiography (MRA) as stroke mimics (eg todd paralysis / complicated
migraine )and etio of stroke in peds is varied than adults - high sensitive test
reqd therefore

MRI/MRA is more sensitive than CT scan - SO CT MAY BE NORMAL THATS WHY MRI
WITH MRA

use of serum prolactin in seizures ?


elevated immediately following seizure

Nervous system -DONE 83


used primarily to differentiate seizure from pseudoseizure (psychogenic nonepileptic seizure)
in adults

not recommended in the routine evaluation of patients with suspected Acute Ischemic Stroke

childhood phenotype of myotonic dystrophy


presents by age 10 - asso with longer CTG repeats and presents with ?

1.cognitive (eg, intellectual impairment)

2.behavioral manifestations (eg, ADHD, mood disorder)

3.excessive daytime sleepiness

Prognosis of Myotonic Dystrophy?


Respiratory muscle failure and cardiac arrhythmias are responsible for most deaths (average
age 45-55).

Crustations in external auditary canal . otalgia. no fever and same side


facial palsy dx ?
vesicular rash with crustations > reactivation of VZV virus in geniculate ganglion

after primary VZV , virus then migrates via sensory nerves to cranial nerve and dorsal spinal
ganglia, where it lies dormant for years

Pneumococcus / invasive pseudomonas VS VZV reactivation


both cause cranial nerve palsies

Rash ? pneumo ( no vesicular rash ) , pseudo ( granulation tissue in EAC) VZV ( vesicular
rash with crustations )

Fever ? present ( absent )

Can invasive Pseudomonas infection be seen in children?


NO, typically presents in elderly, immunocompromised (eg, poorly controlled diabetes)

Polio vs Guillain-Barré syndrome (GBS)


Both have lower motor neuron s/s

weakness ? - asymmetric ( symmetric , ascending )

preceded by ? - aseptic meningitis eg, fever, headache, fatigue ( respi/GI mucosal infection)

Cerebral arteriovenous malformations vs carotid artery dissection


onset & h/o? spontaneous without antecedent trauma ( h/o of Posterior oropharyngeal injuries
/ neck trauma )

Posterior oropharyngeal injuries can result in internal carotid artery dissection or thrombus
formation

Nervous system -DONE 84


Subgaleal hemorrhage is caused by?
the rupture of emissary veins between the dural sinuses and scalp due to scalp traction during
delivery

How does subgaleal hemorrhage present physically ?

diffuse, fluctuant scalp swelling. - extends beyond the suture lines and potentially
beyond the skull to the neck. - swelling shifts with movement and expands over 2-3
days

complication of subgaleal hemorrhage ?


Rapid hemorrhage expansion can lead to hypovolemic shock, DIC and death

signs of blood loss (eg, pallor, anemia, tachycardia),

Massive Blood accumulates between the periosteum and galea aponeurotica - due to
shearing of veins that connect dural sinus to scalp

Does subgaleal hemorrhage occur only with vacuum-assisted deliveries?


NO, can also occur with spontaneous vaginal or cesarean delivery

Mx of subgaleal hemorrhage ?

Treatment is generally supportive (eg, volume resuscitation, correction of coagulopathy).

C/O to subdural , epidural hematoma - no fluctuant swelling of scalp

Cephalohematoma vs Subgaleal hemorrhage


bleeding between ? skull and periosteum due to subperiosteal vessel rupture ( shearing of
veins between the dural sinuses and scalp)

phys. ex ? firm, nonfluctuant swelling that does not cross suture lines ( diffuse, fluctuant scalp
swelling, crosses suture lines )

blood loss ? - not significant ( very signi, acute anemia )

multiple endocrine neoplasia type 2b . Eye findings ?


conjunctival or corneal neuromas

Does light brown patch occur in sturge-weber syndrome ?

NO , it presents with a facial port-wine stain -red-purple vascular patch.

most common lesions in Von Hippel-Lindau disease?

CNS (eg, cerebellum, spinal cord) and retinal hemangioblastomas

Which infants are prone to cephalohematoma ?


Infants delivered with assistance by forceps or vacuum are at increased risk.

Onset ? Mx of Cephalohematoma ?

Nervous system -DONE 85


Due to slow nature of the bleed, cephalohematomas present hours after birth

observation as resorb spontaneously within a month

increased risk for hyperbilirubinemia and may require phototherapy as blood breaks down

firm, nontender, nonfluctuant scalp swelling, does not cross suture lines

positional plagiocephaly vs cephalohematoma ?


swelling ? absent , just abnormal skull flattening - ( present , firm , non tender , non fluctuant
scalp swelling )

onset ? in young infants ( newborns )

Physiologic chorea of infancy . presentation ?


benign, chorea-like movements in normal newborns

resolves by age 8 months

Cerebral palsy greatest risk factor ?


prematurity (ie, <37 weeks gestation)

Cerebral palsy in premature infants is asso with ?

periventricular leukomalacia (white matter necrosis from ischemia)

intraventricular hemorrhage (germinal matrix bleeding due to fragile vasculature).

Hypertonia and hyperreflexia > predispose to increased risk for foot deformities

Myotonic dystrophy vs cerebral palsy


muscle tone ? - myotonia ie impaired grip release ( sustained hypertonia)

Hyperreflexia ? - absent ( present )

what should be performed in all children with language delay ?


Audiologic testing - to assess for underlying hearing loss

Cause of glaucoma in sturge weber syndrome ?


due to a congenital anterior chamber angle anomaly and increased episcleral venous
pressure (from episcleral hemangioma).

eye ipsilateral to the cutaneous lesion is usually affected

surgery is the primary treatment for pediatric glaucoma to preserve vision.

Pt with normal neurological examination . Recurrent U/L headaches with


intermittent nausea vomiting . Headache exacerbated by physical activity .
NBSIM ?

Migraine - Dx clinically , can be exacerbated by physical activity,

Nervous system -DONE 86


When is alternate dx considered in case of migraine like s/s ? x3
1.Abnormal neurologic examination (eg, weakness, ataxia)

2. signs of increased ICP ( headache wakes from sleep, papilledema )

3.rapid increase in headache freq

Simple febrile seizure
TYPE ? - GTCS

DURATION ? - <15 minutes.

Course ? - do not recur within 24 hours.

Antipyretics do not reduce the risk of future febrile seizures.

For simple febrile seizures , neither EEG nor antiseizure medication is recommended.

Life expectancy in Fragile X syndrome pts ?


normal life expectancy

How does Fragile X syndrome present in infants and young children ?


hypotonia and macrocephaly

joint hypermobility can also occur

speech and motor delays

Hypoplastic fingers/nails and cleft lip/palate . Dx ?


fetal hydantoin syndrome - due to phenytoin intake in last trimester

Pregnant women - receive prophylactic vitamin K to prevent neonatal bleeding as phenytoin


may increase the rate of fetal vitamin K degradation

Cerebral contusion vs epidural hematoma


symptom of increased ICP onset ? - develops gradually over hours to days (vs minutes).

Findings in cerebral contusion ?

microhemorrhages ("bruising") within the superficial brain parenchyma due to coup-


contrecoup injury.

when is USG useful for examining head in children ?


requires a widely open anterior fontanelle and is most useful in infants under 6 months of age.

Predominant finding in pts with concussion ?


1.headache - shortly after head trauma - triggered by physical / cognitive exertion / visual
tasks eg eye movements

2. Light and/or noise sensitivity - worsens headache

Nervous system -DONE 87


3.emotional changes (eg irritability), and sleep disturbances.

Secondary dermatomal VZV reactivation . Precautions reqd ?


airborne isolation or contact precautions are not required for immunocompetent
patients who have secondary dermatomal VZV reactivation

only standard precautions

why is isolation precautions done for primary VZV ?


primarily due to the prolonged suspension of viral particles in the air.- so airborne precautions
must

transmission of VZV can occur through contact of skin but VZV doesnt survive for long on hard
surfaces

Lyme arthritis vs Transient synovitis


ESR - elevated ( mild /normal)

Joint involved , pain ?recurrent joint (most commonly knee) pain, esp knee , swollen ( hips ,
mild effusion or no eff , constant pain )

arthrocentesis - round 50 K wbc ( normal )

Juvenile myoclonic epilepsy


presents in adolescents with myoclonic jerks immediately on wakening.

Absence and generalized tonic-clonic seizures may also be seen

How does Lennox-Gastaut syndrome present ?


presents by age 5 with intellectual disability

and severe seizures of varying types (eg, atypical absence, tonic)

EEG findings in Lennox-Gastaut syndrome?


Interictal EEG demonstrates a slow spike-and-wave pattern.

Focal seizures vs Absence seizures


initial involvment of cerebral hemi ? single , might spread to both ( both involved )

Post ictal phase - present ( absent )

Provoked by hyperventilation ? no ( yes )

Nervous system -DONE 88

You might also like